Как сложить два вектора: Страница не найдена — формулки.ру

Содержание

Сумма нескольких векторов

Материал урока.

Вам уже известны правила сложения и вычитания двух векторов.

Чтобы сложить два неколлинеарных вектора  и  по правилу треугольника, нужно от некоторой точки А отложить вектор , равный вектору . Далее от точки B отложить вектор , равный вектору . Вектор  является вектором суммы двух векторов  и .

Чтобы сложить два вектора по правилу параллелограмма, нужно отложить от произвольной точки А векторы  и , равные векторам  и  соответственно, и построить на них параллелограмм ABCD. Тогда вектор  равен сумме векторов  и .

Также вам уже знакомы законы сложения векторов: переместительный и сочетательный.

Ну, а убедившись в том, что разность векторов  и  равна сумме вектора  и вектора, противоположного вектору , мы получили два способа построения вектора разности двух векторов.

Сегодня мы будем учиться складывать несколько векторов в пространстве. Но сначала вспомним, как мы это делали на плоскости.

Построим вектор суммы векторов ,  и .

От некоторой точки А отложим вектор , равный вектору . Далее от точки B отложим вектор , равный вектору . А от точки C отложим вектор , равный вектору .

Будем последовательно складывать наши векторы, пользуясь правилом треугольника.

Сумма векторов  и   равна вектору .

Теперь к вектору  добавим вектор . В результате мы получаем вектор .

Тогда можем сказать, что сумма векторов ,  и . равна вектору .

 

Так, последовательно складывая первый вектор со вторым, затем их сумму с третьим и так далее, можно найти суммы четырёх, пяти и большего числа векторов.

Такое правило построения суммы векторов называют

правилом многоугольника, и оно позволяет построить вектор суммы неограниченного количества векторов.

Задача. Построить вектор суммы попарно неколлинеарных векторов , , ,  и .

Построение.

 

Примеры, приведённые нами, подходят для векторов, лежащих в одной плоскости. А мы, изучая стереометрию, находимся в пространстве, поэтому правило многоугольника сложения векторов в пространстве может иметь и другую иллюстрацию.

 

Задача. Рассмотрим векторы ,  и , такие, что ,  лежат в одной плоскости, а вектор  не лежит в этой плоскости. Найдём сумму этих векторов.

Решение.

Выберем любую удобную точку О в пространстве и отложим от неё вектор , равный вектору , а от точки А отложим вектор , равный вектору . Понятно, что через проведённые векторы можно провести плоскости. Далее, от точки B отложим вектор , равный вектору . Вектором суммы данных векторов является вектор .

Вы видите, что многоугольник сложения в данном случае является пространственным, то есть не все его вершины лежат в одной плоскости.

Сформулируем правило многоугольника для произвольных точек пространства А1, А2 ,…, Аn.

Это равенство справедливо для любых точек А1, А2, …, An. И, в частности, для случая, когда некоторые из них совпадают.

Например, если начало первого вектора совпадает с концом последнего, то сумма данных векторов равна .

Задача. Упростить выражения

Выполним задание, где, пользуясь данной формулировкой, упростим выражения.

а)

б)

в)

г) =

Так мы с вами рассмотрели примеры преобразования выражений с векторами, представленных в виде алгебраической суммы.

Задача. , , ,  произвольные точки пространства.

Представить вектор  в виде алгебраической суммы векторов:

а) , ,                            б) , ,                             в) , ,

Решение.

В последнем задании рассмотрим параллелепипед ABCDA1B1C1D1.

 Нужно указать вектор , начало и конец которого являются вершинами параллелепипеда. И чтобы истинными были данные равенства.

Сумма векторов .

По рисунку понятно, чтобы восстановить правило многоугольника, не достает вектора . Значит, вектор .

Далее рассмотрим выражение, где сумма векторов .

 

По рисунку понятно, что сумма известных векторов из левой части равенства равна вектору . И чтобы вся сумма равнялась вектору , вектор  должен быть равен вектору .

Перейдём к следующему равенству.

Чтобы восстановить правило многоугольника, вектор  удобнее заменить равным ему вектором . Тогда становится понятно, что вектор «-» равен вектору . А вектор  отсюда равен вектору .

Разберёмся с последним равенством. .

Левую часть представим в виде суммы и заменим вектор «– » на .

Изобразим данные векторы. Видим, что искомый вектор  равен вектору .

Подведём итоги урока.

Сегодня мы сформулировали правило многоугольника сложения нескольких векторов в пространстве. И нашли его отличие от того же правила на плоскости.

Оно заключается в том, что полученный многоугольник может являться пространственным, то есть не все его вершины лежат в одной плоскости.

Также мы сформулировали правило многоугольника для произвольных точек пространства А1, А2 …, Аn.

Сумма векторов + ,+ =  ,.

И если начало первого вектора совпадает с концом последнего, то сумма данных векторов равна .

Эти знания мы смогли применить при выполнении заданий.

Сложение векторов. Векторная сумма. Правила сложения векторов. Геометрическая сумма. Он-лайн калькулятор.





Адрес этой страницы (вложенность) в справочнике dpva.ru:  главная страница  / / Техническая информация / / Математический справочник / / Линейная алгебра. Вектора, матрицы, определители, миноры, детерминанты…  / / Сложение векторов. Векторная сумма. Правила сложения векторов. Геометрическая сумма. Он-лайн калькулятор.

Поделиться:   

Сложение векторов. Векторная сумма. Правила сложения векторов. Геометрическая сумма. Он-лайн калькулятор

В механике существуют два типа величин:
  • скалярные величины, задающие некоторое числовое значение — время, температура, масса и т.д.
  • векторные величины, которые вместе с некоторым числовым значением задают направление — скорость, сила и т.д..
Рассмотрим сначала алгебраический подход к сложению векторов.

Покоординатное сложение векторов.

Пусть даны два вектора, заданные покоординатно ( чтобы вычислить координаты вектора, нужно вычесть из соответствующих координат его конца соответствующие координаты его начала, т.е. из первой координаты — первую, из второй — вторую и т.д.):

Тогда координаты вектора, получившегося при сложении этих двух векторов вычисляются по формуле:

В двумерном случае все абсолютно анологично, просто отбрасываем третью координату.

Теперь перейдем к геометрическому смыслу сложения двух векторов:

При сложении векторов нужно учитывать и их числовые значения, и направления. Есть несколько широко используемых методов сложения:

  • правило параллелограмма
  • правило треугольника
  • тригонометрический способ

Правило параллелограмма. Сложение векторов по правилу параллелограмма.

Процедура сложения векторов по правилу параллелограмма заключается в следующем:

  • нарисовать первый вектор, учитывая его величину и направление
  • от начала первого вектора нарисовать второй вектор, также используя и его величину, и его направление
  • дополнить рисунок до параллелограмма, считая, что два нарисованных вектора — это его стороны
  • результирующим вектором будет диагональ параллелограмма, причем его начало будет совпадать с началом первого (а, значит, и второго) вектора.

Правило треугольника. Сложение векторов по правилу треугольника.

Сложение векторов по правилу треугольника заключается в следующем:

  • нарисовать первый вектор, используя данные о его длине ( числовой величине) и направлении
  • от конца первого вектора нарисовать второй вектор, также учитывая и его размер, и его направление
  • результирующим вектором будет вектор, начало которого совпадает с началом первого вектора, а конец — с концом второго.

Тригонометрический способ. Сложение векторов тригонометрическим способом.

Результирующий вектор сложения двух компланарных векторов может быть вычислен с помощью теоремы косинусов:
  • Fрез. = [ F12 + F22 -2 F1 F2 cos(180о-α) ]1/2         (1)
    • где
      • F = числовое значение вектора
      • α = угол между векторами 1 и 2
Угол между результирующим вектором и одним из исходных векторов может быть вычислен по теореме синусов:
  • β = arcsin[ F*sin(180o-α) / FR ]         (2)
    • где
      • α = угол между исходными векторами

Пример — сложение векторов.

Сила 1 равна 5кН и воздействует на тело в направлении, на 80o отличающемся от направления действия второй силы, равной 8 кН.

Результирующая сила вычисляется следующим образом:

Fрез = [ (5 кН)2 + (8 кН)2 — 2 (5 кН)(8 kН) cos(180o — (80o)) ]1/2

    = 10,14кН

Угол между результирующей силой и первой силой равен:

β= arcsin[ (8кН) sin(180o — (80o)) / (10,14кН)]

    = 51o

А угол между второй и результирующей силой можно посчитать следующим образом: as

α = arcsin [ (5 кН) sin(180o — (80o)) / (10,2 кН)]

    = 29o

Он-лайн калькулятор сложения векторов.

Калькулятор ниже может быть использован для любвых векторных величин ( силы, скорости и т.д.) Точка начала вектора совпадает с началами обоих исходных векторов.

Поиск в инженерном справочнике DPVA. Введите свой запрос:
Поиск в инженерном справочнике DPVA. Введите свой запрос:
Если Вы не обнаружили себя в списке поставщиков, заметили ошибку, или у Вас есть дополнительные численные данные для коллег по теме, сообщите , пожалуйста.
Вложите в письмо ссылку на страницу с ошибкой, пожалуйста.
Коды баннеров проекта DPVA.ru
Начинка: KJR Publisiers

Консультации и техническая
поддержка сайта: Zavarka Team

Проект является некоммерческим. Информация, представленная на сайте, не является официальной и предоставлена только в целях ознакомления. Владельцы сайта www.dpva.ru не несут никакой ответственности за риски, связанные с использованием информации, полученной с этого интернет-ресурса. Free xml sitemap generator

Операции с векторами, сложение векторов, умножение вектора на действительное число.

Рассмотрим вектор v с начальной точкой в начале координат в любой координатной системе x-y и с конечной точкой в (a,b). Мы говорим, что вектор находится в стандартном положении и ссылаемся на него как на радиус-вектор. Обратите внимание, что пара точек определяет этот вектор. Таким образом, мы можем использовать это для обозначения вектора. Чтобы подчеркнуть, что мы имеем в виду вектор, и, чтобы избежать путаницы, как правило, пишут:
v = .


Координата a есть скаляром горизонтальной компоненты вектора, и координата b есть скаляром вертикальной компоненты вектора. Под скаляром мы подразумеваем численное количество, а не векторную величину. Таким образом, это рассматривается как компонентная форма v. Обратите внимание, что a и b НЕ вектора и их не надо путать с определением компонента вектора.

Теперь рассмотрим с A = (x1, y1) и C = (x2, y2). Давайте рассмотрим, как найти радиус вектор, эквивалентный . Как Вы видите на рисунке внизу, начальная точка A перемещена в начало координат (0, 0). Координаты P находятся вычитанием координат A из координат C. Таким образом, P = (x2 — x1, y2 — y1) и радиус вектор есть .

Можно показать, что и имеют одну и ту же величину и направление, и поэтому эквивалентны. Таким образом, = = 2 — x1, y2 — y1 >.

Компонентная форма с A = (x1, y1) и C = (x2, y2) есть
= 2 — x1, y2 — y1 >.

Пример 1 Найдите компонентную форму если C = (- 4, — 3) и F = (1, 5).

Решение Мы имеем
= = .

Обратите внимание, что вектор есть равным радиус-вектору , как показано на рисунке вверху.

Теперь, когда мы знаем, как записать вектор в компонентной форме, давайте изложим некоторые определения.
Длину вектора v легко определить, когда известны компоненты вектора. Для v = 1, v2 >, мы имеем
|v|2 = v21 + v22          Используя теорему Пифагора
|v| = √v21 + v22.

Длина, или величина ветктора v = 1, v2 > находится как |v| = √v21 + v22.

Два вектора равны или эквивалентны, если они имеют одну и ту же величину и одно и то же направление.

Пусть u = 1, u2 > и v = 1, v2 >. Tогда
1, u2 > = 1, v2 >          только если u1 = v1 and u2 = v2.

Операции с векторами

Чтобы умножить вектор V на положительное число, мы умножаем его длину на это число. Его направление остается прежним. Когда вектор V умножается на 2, например, его длина увеличивается в два раза, но его направление не изменяется. Когда вектор умножается на 1,6, его длина увеличивается на 60%, а направление остается прежним. Чтобы умножить вектор V на отрицательное действительное число, умножаем его длину на это число и изменяем направление на противоположное. Например, Когда вектор умножается на (-2), его длина увеличивается в два раза и его направление изменяется на противоположное. Так как действительные числа работают как скалярные множители в умножении векторов, мы называем их скаляры и произведение kv называется скалярные кратные v.

Для действительного числа k и вектора v = 1, v2 >, скалярное произведение k и v есть
kv = k.1, v2 > = 1, kv2 >.
Вектор kv есть скалярным кратным вектора v.

Пример 2 Пусть u = и w = . Найдите — 7w, 3u и — 1w.

Решение
— 7w = — 7. = ,
3u = 3. = ,
— 1w = — 1. = .

Теперь мы можем сложить два вектора, используя компоненты. Чтобы сложить два вектора в компонентной форме, мы складываем соответствующие компоненты. Пусть u = 1, u2 > и v = 1, v2 >. Тогда
u + v = 1 + v1, u2 + v2 >

Например, если v = и w = , тогда
v + w = =

Если u = 1, u2 > и v = 1, v2 >, тогда
u + v = 1 + v1, u2 + v2 >.

Перед тем, как мы определим вычитание векторов нам нужно дать определение — v. Противоположный вектору v = 1, v2 >, изображенному внизу, есть
— v = (- 1).v = (- 1)1, v2 > = 1, — v2 >

Вычитание векторов, такое как u — v вовлекает вычитание соответствующих компонент. Мы покажем это представлением u — v как u + (- v). Если u = 1, u2 > и v = 1, v2 >, тогда
u — v = u + (- v) = 1, u2 > + 1, — v2 > = 1 + (- v1), u2 + (- v2) > = 1 — v1, u2 — v2 >

Мы можем проиллюстрировать вычитание векторов с помощью параллелограмма , как мы это делали для сложения векторов.

Вычитание векторов

Если u = 1, u2 > и v = 1, v2 >, тогда
u — v = 1 — v1, u2 — v2 >.

Интересно сравнить суммы двух векторов с разницей тех же двух векторов в одном параллелограмме. Векторы u + v и u — v есть диагоналями параллелограмма.

Пример 3 Сделайте следующие вычисления, где u = и v = .
a) u + v
b) u — 6v
c)3u + 4v
d)|5v — 2u|

Решение
a) u + v = + = = ;
b)u — 6v = — 6. = — = ;
c) 3u + 4v = 3. + 4. = + = ;
d) |5v — 2u| = |5. — 2.| = | — | = || = √(- 29)2 + 212 = √1282 ≈ 35,8

Прежде чем сформулировать свойства векторного сложения и умножения, мы должны дать определение еще одному специальному вектору — нулевому вектору. Вектор, чья начальная точка совпадает с конечной точкой, называется нулевым вектором, обозначается O, или . Его величина равна 0. В сложении векторов:
v + O = v.          1, v2 > + = 1, v2 >
Операции над векторами обладают те же самыми свойствами, что и операции над вещественными числами.

Свойства векторного сложения и умножения

Для всех векторов u, v, и w, и для всех скаляров b и c:
1. u + v = v + u.
2. u + (v + w) = (u + v) + w.
3. v + O = v.
4 1.v = v;          0.v = O.
5. v + (- v) = O.
6. b(cv) = (bc)v.
7. (b + c)v = bv + cv.
8. b(u + v) = bu + bv.

Орты

Вектор величиной, или длиной 1 называется орт. Вектор v = есть орт, потому что
|v| = || = √(- 3/5)2 + (4/5)2 = √9/25 + 16/25 = √25/25 = √1 = 1.

Пример 4 Найдите орт, который имеет то же самое направление, что и вектор w = .

Решение Найдем сначала длину w:
|w| = √(- 3)2 + 52 = √34. Таким образом, мы ищем вектор, с длиной 1/√34 от w и с таким же самым направлением, что и вектор w. Этот вектор есть
u = w/√34 = /√34 = 34, 5/√34 >.
Вектор u есть орт, потому что
|u| = |w/√34| = = √34/34 = √1 = 1.

Если v есть вектор и v ≠ O, тогда
(1/|v|)• v,          or          v/|v|,
есть орт в направлении v.

Хотя орты могут иметь любое направление, орты, параллельные осям x и y особенно полезны. Они определяются как
i =          and          j = .

Любой вектор может быть выражен как линейная комбинация орта i и j. Например, пусть v = 1, v2 >. Tогда
v = 1, v2 > = 1, 0 > + 2 > = v1 + v2 = v1i + v2j.

Пример 5 Выразите вектор r = как линейную комбинацию i и j.

Решение
r = = 2i + (- 6)j = 2i — 6j.

Пример 6 Запишите вектор q = — i + 7j в компонентной форме.

Решениеq = — i + 7j = -1i + 7j =

Векторные операции могут быть также выполнены, когда векторы записаны как линейные i и j.

Пример 7 Если a = 5i — 2j и b = -i + 8j, найдите 3a — b.

Решение
3a — b = 3(5i — 2j) — (- i + 8j) = 15i — 6j + i — 8j = 16i — 14j.

Углы обзора

Конечная точка P орты в стандартной позиции есть точкой на единичной окружности, определенной (cosθ, sinθ). Таким образом, орт может быть выражен в компонентной форме,
u = ,
или как линейная комбинация орт i и j,
u = (cosθ)i + (sinθ)j,
где компоненты u есть функциями угла обзора θ измеряемого против часовой стрелки от оси x к этому вектору. Так как θ изменяется от 0 до 2π, точка P отслеживает круг x2 + y2 = 1. Это охватывает все возможные направления ортов и тогда уравнение u = (cosθ)i + (sinθ)j описывает каждый возможный орт на плоскости.

Пример 8 Вычислите и сделайте эскиз орта u = (cosθ)i + (sinθ)j для θ = 2π/3. Изобразите единичную окружность на эскизе.

Решение
u = (cos(2π/3))i + (sin(2π/3))j = (- 1/2)i + (√3/2)j

Пусть v = 1, v2 > с углом обзора θ. Используя определение функции тангенса, мы можем определить угол обзора их компонент v:

Пример 9 Определите угол обзора θ вектора w = — 4i — 3j.

Решение Мы знаем, что
w = — 4i — 3j = .
Таким образом, имеем
tanθ = (- 3)/(- 4) = 3/4          и θ = tan— 1(3/4).
Так как w находится в третьем квадранте, мы знаем, что θ есть углом третьего квадранта. Соответствующий угол есть
tan— 1(3/4) ≈ 37°,          и          θ ≈ 180° + 37°, или 217°.

Это удобно для работы с прикладными задачами, а в последующих курсах, чтобы иметь способ выразить вектор так, чтобы его величина и направление могли быть легко определены или прочитаны. Пусть v это вектор. Тогда v/|v| есть орт в том же самом направлении, что и v. Таким образом, мы имеем
v/|v| = (cosθ)i + (sinθ)j
v = |v|[(cosθ)i + (sinθ)j]              Умножая на |v|
v = |v|(cosθ)i + |v|(sinθ)j.

Углы между векторами

Когда вектор умножается на скаляр, результатом есть вектор. Когда складываются два вектора, результатом также есть вектор. Таким образом, мы могли бы ожидать, что произведение двух векторов есть вектор, но это не так. Скалярное произведение двух векторов есть действительное число или скаляр. Этот результат полезен в нахождении угла между двумя векторами и в определении, являются ли два вектора перпендикулярными.

Скалярное произведение двух векторов u = 1, u2 > и v = 1, v2 > is
u • v = u1.v1 + u2.v2
(Обратите внимание, что u1v1 + u2v2 есть скаляром, а не вектором.)

Пример 10Найдите скалярное произведение, когда
u = , v = и w = .
a)u • w
b)w • v

Решение
a) u • w = 2(- 3) + (- 5)1 = — 6 — 5 = — 11;
b) w • v = (- 3)0 + 1(4) = 0 + 4 = 4.

Скалярное произведение может быть использовано для нахождения угла между двумя векторами. Угол между двумя векторами это самый маленький положительный угол, образованный двумя направленными отрезками. Таким образом, θ между u и v это тот же самый угол, что и между v и u, и 0 ≤ θ ≤ π.

Если θ есть углом между двумя ненулевыми векторами u и v, тогда
cosθ = (u • v)/|u||v|.

Пример 11Найдите угол между u = и v = .

Решение Начнем с нахождения u • v, |u|, и |v|:
u • v = 3(- 4) + 7(2) = 2,
|u| = √32 + 72 = √58, and
|v| = √(- 4)2 + 22 = √20.
Tогда
cosα = (u • v)/|u||v| = 2/√58.√20
α = cos— 1(2/√58.√20)
α ≈ 86,6°.

Равновесие сил

Когда несколько сил действуют на одну и ту же точку на объекте, их векторная сумма должна быть равна нуля, для того, чтобы был баланс. Когда есть баланс сил, то объект является стационарным или движется по прямой линии, без ускорения. Тот факт, что векторная сумма должна быть равна нулю вывода для получения баланса, и наоборот, позволяет решать нам многие прикладные задачи с участием сил.

Пример 12 Подвесной блок 350- фунтовый блок подвешен с помощью двух кабелей. осталось. В точке А есть три силы, действующие так: W блок тянет вниз, а R и S (два кабеля) тянут вверх и наружу. Найдите нагрузку каждого кабеля.

Решение Нарисуем диаграмму с начальными точками каждого вектора в начале кооординат. Для баланса, сумма векторов должна быть равна О:

R + S + W = О.
Мы можем выразить каждый вектор через его величину и угол обзора :
R = |R|[(cos125°)i + (sin125°)j],
S = |S|[(cos37°)i + (sin37°)j], и
W = |W|[(cos270°)i + (sin270°)j]
= 350(cos270°)i + 350(sin270°)j
= -350j          cos270° = 0; sin270° = — 1.
Заменяя R, S, и W in R + S + W + O, мы имеем
[|R|(cos125°) + |S|(cos37°)]i + [|R|(sin125°) + |S|(sin37°) — 350]j = 0i + 0j.
Это дает нам систему уравнений:
|R|(cos125°) + |S|(cos37°) = 0,
|R|(sin125°) + |S|(sin37°) — 350 = 0.
Решая эту систему, мы получаем
|R| ≈ 280 и |S| ≈ 201.
Таким образом, нагрузка на кабели 280 фунтов и 201 фунт.

Сумма двух векторов. Законы сложения векторов. Сумма нескольких векторов. Правило параллелограмма. Вычитание векторов 9

Тема 24.

Сумма векторов. Разность векторов.

Рассмотрим пример. Пусть материальная точка переместилась из точки A в точку B, а затем из точки B в точку C. В результате этих перемещений, которые можно представить векторами AB⃗ и BC⃗, материальная точка переместилась из точки A в точку C. Поэтому результирующее перемещение можно представить вектором AC⃗. Поскольку перемещение из точки A в точку C складывается из перемещения из A в B и перемещения из B в C, то вектор AC⃗ естественно назвать суммой векторов AB⃗ и BC⃗:AC⃗=AB⃗+BC⃗.

Рассмотренный пример приводит нас к понятию суммы двух векторов.

Пусть a⃗ и b⃗ – два вектора. Отметим произвольную точку A и отложим от этой точки вектор AB⃗ равный a⃗. Затем от точки B отложим вектор BC⃗, равный b⃗. Вектор AC⃗ называется суммой векторов a⃗ и b⃗. Это правило сложения векторов называется правилом треугольника. Рисунок это поясняет.

Сумма векторовa⃗ и b⃗ обозначается так: a⃗+b⃗.

Складывая по правилу треугольника произвольный вектор a⃗ с нулевым вектором, получаем, что для любого вектора a⃗ справедливо равенство

a⃗+0⃗=a⃗

Правило треугольника можно сформулировать также следующим образом: если A, B и C – произвольные точки, то AB⃗+BC⃗=AC⃗.

Это равенство справедливо для произвольных точек A, B и C, в частности, в том случае, когда две из них или даже все три совпадают.

Теорема

Для любых векторов a⃗,b⃗ и c⃗ справедливы равенства:

1. a⃗+b⃗=b⃗+a⃗ (переместительный закон).

2. a⃗+b⃗+c⃗=a⃗+b⃗+c⃗ (сочетательный закон).

Докажем первое равенство. Рассмотрим случай, когда векторы a⃗ и b⃗ не коллинеарны. От произвольной точки A отложим векторы ABAD и на этих векторах построим параллелограмм ABCD. По правилу треугольника AC⃗=AB⃗+BC⃗=a⃗+b⃗. Аналогично AC⃗=AD⃗+DC⃗=b⃗+a⃗. Отсюда следует, что a⃗+b⃗=b⃗+a⃗.

При доказательстве первого свойства мы обосновали так называемое правило параллелограмма сложения неколлинеарных векторов: чтобы сложить неколлинеарные векторы a⃗ и b⃗, нужно отложить от какой-нибудь точки A векторы AB⃗=a⃗ и AD⃗=b⃗ и построить параллелограмм ABCD. Тогда вектор AC⃗ равен a⃗+b⃗. Правило параллелограмма часто используется в физике, например при сложении двух сил.

Сложение нескольких векторов производится следующим образом: первый вектор складывается со вторым, затем их сумма складывается с третьим вектором и т.д. Из законов сложения векторов следует, что сумма нескольких векторов не зависит от того, в каком порядке они складываются. Например, от произвольной точки A отложен вектор AB⃗=a⃗, затем от точки B отложен вектор BC⃗=b⃗ и, наконец, от точки С отложен вектор CD⃗=c⃗. В результате получается вектор AD⃗=a⃗+b⃗+c⃗.

Аналогично можно построить сумму четырех, пяти и вообще любого числа векторов. Это правило построения суммы нескольких векторов называется правилом многоугольника.

Разностью векторов a⃗ и b⃗ называется такой вектор, сумма которого с вектором b⃗ равна вектору a⃗.

Разность векторов a⃗ и b⃗ обозначается так:a⃗-b⃗.

Рассмотрим задачу о построении двух векторов.

Даны векторы a⃗ и b⃗. Построить вектор a⃗-b⃗.

Отметим на плоскости произвольную точку O и отложим от этой точки векторы OA⃗=a⃗ и OB⃗=b⃗.

По правилу треугольника OB⃗+BA⃗=OA⃗ или b⃗+BA⃗=a⃗. Таким образом, сумма векторов BA⃗ и b⃗ равна a⃗. По определению разности векторов это означает, что BA⃗=a⃗-b⃗, то есть вектор BA⃗ искомый.

Пусть a⃗ – произвольный ненулевой вектор. Вектор a1⃗ называется противоположным вектору a⃗, если векторы a⃗ и a1⃗ имеют равные длины и противоположно направлены.

Вектор, противоположный вектору a⃗, обозначается так: -a⃗. Очевидно, что a⃗+-a⃗=0⃗.

Теорема

Для любых векторов a⃗ и b⃗ справедливо равенство a⃗-b⃗=a⃗+-b⃗.

Сегодня мы научились складывать и вычитать векторы. Узнали правило треугольника, правило параллелограмма и правило многоугольника.

Вектор. Правило сложение векторов | Подготовка к ЕГЭ по математике

Здесь рассматриваем вектора на плоскости.

Основные определения

 

Вектором называется направленный отрезок , где точка  – начало, точка  – конец вектора.

Нулевым вектором    называется вектор, у которого начало совпадает с концом.

Векторы  и  называются одинаково направленными или сонаправленными, если лучи AB и CD одинаково направлены.

Если лучи AB и CD противоположно направлены, векторы  и называются противоположно направленными.

 

Два вектора называются коллинеарными , если они лежат на одной прямой или на параллельных прямых.

  

Абсолютной величиной (или модулем) вектора называется длина отрезка, изображающего вектор. Абсолютную величину вектора  обозначают .

Два вектора называются равными, если они одинаково направлены и равны по абсолютной величине.

Два вектора с равными модулями, лежащие на параллельных прямых, но противоположно направленные, называются противоположными. Вектор, противоположный вектору , обозначается как .

Сложение векторов

 

Сложение векторов   и   по правилу треугольника

Суммой   двух векторов   и  называют такой третий вектор , начало которого совпадает с началом , а конец – с концом   при условии, что конец вектора   и начало вектора   совпадают.

Сложение векторов   и   по правилу параллелограмма

Если два неколлинеарных вектора   и   привести к общему началу, то вектор   совпадает с диагональю параллелограмма, построенного на векторах   и . Причем начало вектора  совпадает с началом заданных векторов.

Разностью  векторов  и  называется вектор   такой, что выполняется условие: .

 

Смотрите также «Вектора. Часть 2».

Сложение и вычитание векторов [wiki.eduVdom.com]

Пусть $\overrightarrow{a}$ и $\overrightarrow{b}$ — два вектора (рис.1, а).


Сложение двух векторов

Рис.1

Возьмем произвольную точку О и построим вектор $\overrightarrow{ОА} = \overrightarrow{a}$ . Затем от точки А отложим вектор $\overrightarrow{AB} = \overrightarrow{b}$. Вектор $\overrightarrow{OB}$, соединяющий начало первого слагаемого вектора с концом второго (рис.1, б), называется суммой этих векторов и обозначается $\overrightarrow{a} + \overrightarrow{b}$$ (правило треугольника).

Ту же самую сумму векторов можно получить иным способом. Отложим от точки О векторы $\overrightarrow{ОА} = \overrightarrow{a} \,и\, \overrightarrow{ОС} = \overrightarrow{b} $ (рис.1, в). Построим на этих векторах как на сторонах параллелограмм ОABC. Вектор $\overrightarrow{ОВ}$, служащий диагональю этого параллелограмма, проведенной из вершины О, является, очевидно, суммой векторов $\overrightarrow{a} + \overrightarrow{b}$ {правило параллелограмма). Из рисунка 1, в непосредственно следует, что сумма двух векторов обладает переместительным свойством: $\overrightarrow{a} + \overrightarrow{b} = \overrightarrow{b} + \overrightarrow{a}$

Действительно, каждый из векторов $\overrightarrow{a} + \overrightarrow{b} \,и\, = \overrightarrow{b} + \overrightarrow{a}$ равен одному и тому же вектору $\overrightarrow{OB}$ .



Пример 1.2} = \sqrt{9 + 16} = 5 \\ т.е.\, |\overrightarrow{АВ} + \overrightarrow{ВС}| = 5. $$

Понятие суммы векторов можно обобщить на случай любого конечного числа слагаемых векторов.

Пусть, например, даны три вектора $\overrightarrow{a}, \overrightarrow{b} \,и\, \overrightarrow{c}$ (рис.2).


Сложение трех векторов

Рис.2

Построив сначала сумму векторов $\overrightarrow{a} + \overrightarrow{b}$ , а затем прибавив к этой сумме вектор $\overrightarrow{c}$, получим вектор $(\overrightarrow{a} + \overrightarrow{b}) + \overrightarrow{c}$ . На рисунке 2 $$ \overrightarrow{ОА} = \overrightarrow{a}\,; \overrightarrow{АВ} = b\,; \overrightarrow{ОВ} = \overrightarrow{a} + \overrightarrow{b}\,; \overrightarrow{BC} = \overrightarrow{c} \\ и \\ \overrightarrow{ОС} = \overrightarrow{ОВ} + \overrightarrow{ВС} = (\overrightarrow{a} + \overrightarrow{b}) + \overrightarrow{c} $$ Из рисунка 2 видно, что тот же вектор $\overrightarrow{ОС}$ мы получим, если к вектору $\overrightarrow{ОА} = \overrightarrow{a}$ прибавим вектор $\overrightarrow{АВ} = \overrightarrow{b} + \overrightarrow{c}$ . Таким образом, $(\overrightarrow{a} + \overrightarrow{b}) + \overrightarrow{c} = \overrightarrow{a} + (\overrightarrow{b} + \overrightarrow{c})$ , т. е. сумма векторов обладает сочетательным свойством. Поэтому сумму трех векторов $\overrightarrow{a}\,,\,\overrightarrow{b}\,,\,\overrightarrow{c}$ записывают просто $\overrightarrow{a} + \overrightarrow{b} + \overrightarrow{c}$ .

Разностью двух векторов $\overrightarrow{a} \,и\, \overrightarrow{b}$ называется третий вектор $\overrightarrow{c} = \overrightarrow{a} — \overrightarrow{b}$ , сумма которого с вычитаемым вектором $\overrightarrow{b}$ дает вектор $\overrightarrow{a}$. Таким образом, если $\overrightarrow{c} = \overrightarrow{a} — \overrightarrow{b}\,,\, то\, \overrightarrow{c} + \overrightarrow{b} = \overrightarrow{a}$ .

Из определения суммы двух векторов вытекает правило построения вектора-разности (рис.3).


Вычитание векторов

Рис.3

Откладываем векторы $\overrightarrow{ОА} = \overrightarrow{a} \,и\, \overrightarrow{OB} = \overrightarrow{b}$ из общей точки О. Вектор $\overrightarrow{BA}$ , соединяющий концы уменьшаемого вектора $\overrightarrow{a}$ и вычитаемого вектора $\overrightarrow{b}$ и направленный от вычитаемого к уменьшаемому, является разностью $\overrightarrow{c} = \overrightarrow{a} — \overrightarrow{b}$ . Действительно, по правилу сложения векторов $\overrightarrow{ОВ} + \overrightarrow{ВА} = \overrightarrow{ОА} \text{ , или } \overrightarrow{b} + \overrightarrow{c} = \overrightarrow{a}$ .


Пример 2. Сторона равностороннего треугольника ABC равна а. Найти: $а) |\overrightarrow{ВА} — \overrightarrow{ВС}|\,;\,\ б)\,\,\ |\overrightarrow{АВ} — \overrightarrow{АС}|$ .

Решение а) Так как $\overrightarrow{ВА} — \overrightarrow{ВС} = \overrightarrow{СА}\text{ , а }|\overrightarrow{СА}| = а\text{ , то }|\overrightarrow{ВА} — \overrightarrow{ВС}| = а$ .

б) Так как $\overrightarrow{АВ} — \overrightarrow{АС} = \overrightarrow{СВ}\text{ , а }|\overrightarrow{СВ}| = а\text{ , то }|\overrightarrow{АВ} — \overrightarrow{АС}| = а$ .

Произведением вектора $\overrightarrow{a}$(обозначается $=\lambda\overrightarrow{a}$ или $\overrightarrow{a}\lambda$) на действительное число $\lambda$ называется вектор $\overrightarrow{b}$, коллинеарный вектору $\overrightarrow{a}$, имеющий длину, равную $|\lambda||\overrightarrow{a}|$, и то же направление, что и вектор $\overrightarrow{a}$, если $\lambda > 0$ , и направление, противоположное направлению вектора $\overrightarrow{a}$, если $\lambda < 0$ . Так, например, $2\overrightarrow{a}$ есть вектор, имеющий то же направление, что и вектор $\overrightarrow{a}$ , а длину, вдвое большую, чем вектор $\overrightarrow{a}$ (рис.4).


Умножение вектора на число

Рис.4

В случае, когда $\lambda = 0$ или $\overrightarrow{a} = 0$ , произведение $\lambda\overrightarrow{a}$ представляет собой нулевой вектор. Противоположный вектор $-\overrightarrow{a}$ можно рассматривать как результат умножения вектора $\overrightarrow{a}$ на $\lambda = -1$ (см. рис.4): $$ -\overrightarrow{a} = \ (-1)\overrightarrow{a} $$ Очевидно, что $\overrightarrow{a} + (-\overrightarrow{a}) = \overrightarrow{0}$ .


Пример 3. Доказать, что если О, А, В и С — произвольные точки, то $\overrightarrow{ОА} + \overrightarrow{АВ} + \overrightarrow{ВС} + \overrightarrow{СО} = 0$ .

Решение. Сумма векторов $\overrightarrow{ОА} + \overrightarrow{АВ} + \overrightarrow{СВ} = \overrightarrow{ОС}$ , вектор $\overrightarrow{CO}$ — противоположный вектору $\overrightarrow{ОС}$ . Поэтому $\overrightarrow{ОС} + \overrightarrow{СО} = \overrightarrow{0}$ .

Пусть дан вектор $\overrightarrow{a}$. Рассмотрим единичный вектор $\overrightarrow{a_0}$ , коллинеарный вектору $\overrightarrow{a}$ и одинаково с ним направленный. Из определения умножения вектора на число следует, что $$ \overrightarrow{a} = |\overrightarrow{a}|\,\ \overrightarrow{a_0} $$ , т.е. каждый вектор равен произведению его модуля на единичный вектор того же направления. Далее из того же определения следует, что если $\overrightarrow{b} = \lambda\overrightarrow{a}$ , где $\overrightarrow{a}$ — ненулевой вектор, то векторы $\overrightarrow{a} \,и\, \overrightarrow{b}$ коллинеарны. Очевидно, что и обратно, из коллинеарности векторов $\overrightarrow{a} \,и\, \overrightarrow{b}$ следует, что $\overrightarrow{b} = \lambda\overrightarrow{a}$.

Таким образом, получаем следующую теорему.


Пример 4. Длина вектора AB равна 3, длина вектора AC равна 5. Косинус угла между этими векторами равен 1/15. Найдите длину вектора AB + AC.

Видео-решение.



Глава 4. Едем по указателям – FIZI4KA

В этой главе…

  • Изучаем сложение и вычитание векторов
  • Выражаем векторы через координаты
  • Разбиваем векторы на компоненты
  • Выражаем перемещение, ускорение и скорость в виде векторов
  • Определяем изменение скорости под действием тяготения

Довольно трудно добраться в место назначения — пешком ли, на велосипеде ли, на автомобиле ли, на самолете ли — если вы не знаете направления движения.2+v_0t \)​. С помощью таких соотношений можно получить значения для ускорения, например 27 метров в секунду в квадрате, или для скорости, например 42,7 мили в час. Конечно, полезно знать эти параметры движения, но что можно сказать о направлении движения?

В реальном мире просто необходимо знать направление движения. Именно векторы обозначают такое направление. Очень многие люди ошибочно считают векторы очень сложными объектами, но это совсем не так. В этой главе вы узнаете, насколько легко и просто можно обращаться с ними при решении задач.

Осваиваем векторы

В главе 3 мы работали с простыми числами или измерениями, которые в физике называются величинами. Например, в результате измерения перемещения на 3 метра получена величина перемещения 3 метра. Вектор отличается от величины еще и наличием направления. В повседневной жизни на вопрос о пути понятие “вектор” возникает в виде следующего ответа встречного человека: “Это в 15 милях отсюда”. При этом величина вектора равна 15 милям, а направление вектора определяется взмахом руки. Когда вы навешиваете дверь на петли, то порой слышите совет: “Толкните сильнее влево”. Вот вам еще один вектор! Когда вы объезжаете препятствие на дороге, вам приходится ускоряться и замедляться в разных направлениях. Вот еще несколько векторов!

Векторы встречаются в обыденных ситуациях, например в дорожных указателях, инструкциях по сборке или даже при попытке избежать столкновения со встречным. Поскольку физика стоит за всеми событиями повседневной жизни, то не удивительно, что многие физические концепции, например скорость, ускорение, сила, являются векторами. По этой причине следует поближе познакомиться с векторами, поскольку они присутствуют во всех разделах физики. Вектор — это фундаментальное понятие физики.

Определяем направление: основные свойства векторов

При работе с векторами нужно иметь в виду его направление и величину. Физический параметр без направления, а только с величиной называется скаляром. Если к скаляру добавить направление, то получим вектор.

Визуально в физических задачах вектор отображается в виде стрелки. Действительно, стрелка имеет величину (т.е. длину) и направление (т.е. острие). Взгляните на рис. 4.1. Эта стрелка и есть вектор с началом в тупом конце и с окончанием — в заостренном конце.

Векторы можно использовать для представления силы, ускорения, скорости и других физических параметров. В физике для обозначения векторов используют полужирное начертание, например A. В некоторых книгах векторы обозначают стрелкой над символом, например ​\( \overrightarrow{A} \)​ . Стрелка обозначает, что у данного параметра ​A​, помимо величины, есть также направление.

Допустим, какой-то умник предложит вам дать пример вектора. Проще простого! Достаточно сказать, что у некого вектора А есть некая величина и некоторое направление. Убежден, что это произведет на умника оглушительное впечатление! Например, скажите, что вектор А направлен под углом 15° к горизонтали и имеет величину 12 метров в секунду. Итак, любопытный умник получит исчерпывающую информацию о векторе А.

На рис. 4.2 показаны два вектора, А и В. Они очень похожи, поскольку обладают одинаковой длиной и направлением. Фактически оба эти вектора равны. Если два вектора равны по величине и направлению, то они считаются равными, т.е. А = В.

Очень скоро читатель станет настоящим экспертом в области векторов. Уже сейчас нам известно, что, когда мы встречаемся с символом А, это значит, что данный параметр обладает величиной и направлением, т.е. является вектором, а два вектора считаются равными, если они имеют одинаковую величину и направление. Но это еще далеко не все. Допустим, чтобы найти нужный вам отель, нужно проехать 20 миль к северу, а потом 20 миль на восток. Так насколько далеко и в каком направлении находится этот отель?

Комбинируем направления: сложение векторов

Два вектора можно сложить и получить результирующий вектор, который является суммой обоих векторов и определяет расстояние и направление до цели.

Допустим, что прохожий говорит вам, что для достижения пункта назначения вам нужно сначала следовать вектору А, а потом вектору В. Так где же находится в этом случае ваш пункт назначения? Сначала нужно проехать по пути, указанному вектором А, а потом по пути, указанному вектором В, как показано на рис. 4.3.

Когда вы доберетесь до конца вектора В, насколько далеко вы будете находиться от исходной точки? Для ответа на этот вопрос начертим еще один вектор С от исходной точки и до конечной точки путешествия, как показано на рис. 4.4.

Новый вектор С представляет собой результат всего путешествия от начала и до самого конца. Все, что нужно сделать, чтобы получить его, так это начертить оба вектора А и В и соединить новым результирующим вектором С.

Сумма векторов достигается за счет того, что начало одного вектора помещается в конец другого, т.е. суммарный вектор проходит от начала одного до конца другого вектора. Иначе говоря, С = А + В. При этом С называется суммой векторов, результатом сложения векторов, или результирующим вектором. Не думайте, что этим ограничиваются возможности комбинирования векторов, ведь векторы можно и вычитать.

Вычисляем разницу расстояний: разность векторов

А что если некто предложит вам векторы С и А, показанные на рис. 4.4, и попросит найти их разность? Их разностью является вектор В, поскольку при сложении векторов А и В получается вектор С. Чтобы объяснить эту мысль, нужно прояснить смысл вычитания вектора А из вектора С: т.е. смысл операции С — А.

Для вычитания двух векторов нужно расположить вместе основания векторов (т.е. концы векторов без остриев), а не совмещать основание одного вектора и острие другого вектора, как при сложении векторов. Затем нужно провести результирующий вектор, который является разностью двух векторов, от острия вычитающего вектора (А) к острию вычитаемого вектора (С). На рис. 4.5 показан пример вычитания вектора А из вектора С (иначе говоря, приведен пример С — А). Как видите, результат такого вычитания равен вектору В, поскольку С = А + В.

Еще один (и для некоторых более простой) способ вычитания векторов заключается в обращении направления второго вектора (т.е. вектора А в разности С — А) и сложении двух векторов: вектора С и обращенного вектора А (т.е. совмещении острия обращенного вектора А с основанием вектора С с последующим проведением результирующего вектора от основания обращенного вектора А к острию вектора С).

Как видите, сложение и вычитание векторов может происходить с одними и теми же векторами в одной задаче. На самом деле с векторами можно выполнять и некоторые другие математические операции. Изложенный выше материал означает, что с векторами можно оперировать так же, как со скалярами, например С = А + В, С — А = В и т.д. Как видите, векторы очень похожи на числа.

Облекаем векторы в числа

Векторы удобно представлять в виде стрелок, но это не всегда самый точный способ работы с ними. Векторы гораздо точнее можно характеризовать числами. Рассмотрим пример сложения векторов А + В, показанных на рис. 4.6.

Предположим, что измерения на рис. 4.6 даны в метрах. Это значит, что вектор А направлен на 1 метр вверх и на 5 метров вправо, а вектор В направлен на 1 метр вправо и на 4 метра вверх. Для получения параметров результирующего вектора С нужно сложить горизонтальные измерения обоих векторов и отдельно сложить вертикальные измерения обоих векторов.

Результирующий вектор С направлен на 6 метров вправо и на 5 метров вверх. Как видите, для получения вертикального измерения вектора С нужно сложить вертикальное измерение вектора А и вертикальное измерение вектора В. А для получения горизонтального измерения вектора С нужно сложить горизонтальное измерение вектора А и горизонтальное измерение вектора В.

Если процедура сложения векторов все еще очень туманна для вас, то тогда можно использовать другую систему обозначений векторов. Поскольку вектор А “простирается” на 5 метров вправо (в положительном направлении оси X) и на 1 метр вверх (в положительном направлении оси Y), то его можно выразить в координатах (х,у), например А = (5;1). Аналогично, поскольку вектор В “простирается” на 1 метр вверх (в положительном направлении оси X) и на 4 метра вправо (в положительном направлении оси Y), то его можно выразить в координатах (х,у), например В = (1;4).

С помощью такой системы обозначений сложение векторов существенно упрощается. Итак, для сложения двух векторов достаточно сложить их координаты по осям X и Y, чтобы получить координаты результирующего вектора по осям X и Y:

Получается, что весь секрет сложения векторов заключается в разбиении каждого вектора на координаты по осям X и Y с последующим их сложением, чтобы соответственно получить координаты X и Y результирующего вектора? Конечно, работа с этими числами для получения координат X и Y результирующего вектора требует некоторых усилий, но они достаточно просты, чтобы с успехом их выполнить.

Допустим, что нужный вам отель находится на расстоянии 20 миль к северу и на расстоянии 20 миль на восток. Как будет выглядеть вектор, направленный из исходной точки к этому отелю? С помощью координатного представления эта задача решается очень легко. Допустим, что положительное направление оси X направлено на восток, а положительное направление оси Y — на север. На первом этапе нужно проехать 20 миль на север, а на втором этапе — 20 миль на восток. В векторном представлении эта задача формулируется следующим образом (восток [X]; север [Y]):

Чтобы сложить эти два вектора, нужно сложить их координаты по соответствующим осям:

Результирующий вектор, который указывает на отель, имеет вид (20; 20).

Рассмотрим еще один пример удачного применения такого представления векторов. Допустим, что вы едете на гоночном автомобиле со скоростью 150 миль в час на восток и видите в зеркало заднего вида приближающегося соперника. Нет проблем, нужно лишь удвоить скорость:

Теперь вы уже не едете, а почти “летите” со скоростью 300 миль в час, но в том же направлении. Итак, в этой задаче демонстрируется процедура умножения вектора на скаляр.

Разбиение вектора на компоненты

Формулировки задач по физике с использованием векторов не всегда так просты, как предыдущие примеры с манипуляциями векторов. Рассмотрим первый вектор на рис. 4.1 с координатами (4; 1) и сравним его со следующей типичной формулировкой физической задачи: найти время перемещения шара со скоростью 7 метров в секунду по наклонной плоскости с длиной основания 1 м, расположенной под углом 15°. С помощью дальнейшей информации в этом разделе вы научитесь находить компоненты векторов и легко и просто манипулировать ими.

Ищем компоненты вектора по заданной величине и углу

Чтобы определить координаты вектора, нужно научиться разбивать векторы на части, которые называются компонентами. Например для вектора (4; 1) Х-компонентой является число 4, а Y-компонентой — число 1.

Часто в физической задаче задается угол и величина вектора, а его компоненты нужно определить. В предыдущем примере известно, что шар катится со скоростью 7 метров в секунду по наклонной плоскости с длиной основания 1 м, расположенной под углом 15°. Для определения времени перемещения шара от одного конца плоскости к другому нам потребуется разобраться только с Х-компонентой. То есть, задача сводится к определению времени перемещения на расстояние 1 метр вдоль оси X. Для ответа на этот вопрос нужно определить скорость перемещения шара по оси X.

Итак, нам известно, что шар движется со скоростью 7 метров в секунду под углом 15° к горизонтали (т.е. положительного направления оси X). В данной формулировке скорость является вектором ​\( \mathbf{v} \)​ с величиной 7 метров в секунду и направлением 15° к горизонтали.

Теперь нам нужно определить Х-компоненту вектора скорости шара, чтобы определить скорость перемещения шара вдоль основания наклонной плоскости. Х-компонента скорости является скаляром (т.е. имеет только значение, а не значение, направление и точку приложения, как вектор) и обозначается как ​\( v_x \)​. Аналогично, Y-компонента скорости шара также является скаляром и обозначается как ​\( v_y \)​. Итак, вектор скорости можно выразить через его компоненты:

Именно так выражается разложение вектора на компоненты. Так чему же равны компонента \( v_x \) и компонента \( v_y \)? Скорость имеет величину ​\( v \)​ (7 метров в секунду) и направление ​\( \theta \)​ (угол 15° к горизонтали). Также нам известна длина основания наклонной плоскости (1,0 метр). На рис. 4.7 показана схема тригонометрических функций (о, Боже, только не это!), которые описывают направление вектора скорости и помогут нам определить его компоненты. Не стоит волноваться: тригонометрические соотношения не так уж и сложны, если известен угол ​\( \theta \)​, показанный на рис. 4.7. Величина (или модуль) вектора \( \mathbf{v} \) равна ​\( v \)​ (иногда если вектор обозначается символом ​\( v \)​, то его модуль обозначают символом ​\( \overline{v} \)​), а его компоненты определяются с помощью рис. 4.7:

Рекомендуется хорошенько запомнить указанные выше выражения для компонент вектора, поскольку нам придется довольно часто встречаться с ними в курсе физики.

Теперь можно пойти немного дальше и попробовать связать отдельные стороны треугольника на рис. 4.7. Это можно легко сделать, если вспомнить соотношение для тангенса \( tg\,\theta=\sin\theta/\cos\theta \) и воспользоваться соотношениями для компонент скорости:

Зная соотношение ​\( v_x=v\cos\theta \)​, можно найти величину Х-компоненты скорости шара \( v_x=v\cos\theta \):

Подставляя числа, получим

Итак, теперь мы знаем, что горизонтальная скорость шара равна 6,7 метров в секунду. Поскольку длина основания наклонной плоскости равна 1,0 метра, то это расстояние шар преодолеет за время:

Таким образом, благодаря тому, что мы научились определять компоненту скорости, нам удалось легко найти решение все задачи: шару потребуется 0,15 секунды для перемещения вдоль наклонной плоскости. А чему равна Y-компонента скорости? Это можно очень легко определить, поступая аналогично:

Находим величину и направление вектора по его компонентам

Иногда требуется определить угол наклона вектора, если известны его компоненты. Например, предположим, что вы ищите отель, расположенный на 20 миль к северу и на 20 миль к востоку. Под каким углом нужно двигаться к нему и насколько далеко он находится? Условия этой задачи можно записать с помощью уже известных нам векторных обозначений (см. предыдущий раздел):

После сложения этих двух векторов получим следующий результат:

Результирующий вектор, который указывает на отель, имеет вид (20; 20). Это еще один способ указания вектора с помощью его компонент. Итак, вернемся к прежнему вопросу: под каким углом нужно двигаться к отелю и насколько далеко он находится от текущего положения? Иначе говоря, глядя на рис. 4.8, прежний вопрос теперь звучит так: “Чему равны ​\( h \)​ и ​\( \theta \)​?”

Найти ​\( h \)​ не так уж и трудно, пользуясь теоремой Пифагора:

Подставляя численные значения, получим:

Итак, отель находится на расстоянии 28,3 мили. А под каким углом ​\( \theta \)​ нужно ехать к нему по прямой? Пользуясь основными тригонометрическими соотношениями, можно записать:

Иначе говоря:

Теперь для определения угла нужно использовать функции, обратные синусу и косинусу:

(Строго говоря, обратной синусу функцией является функция “арксинус”, или ​\( arcsin(x) \)​, а обратной косинусу — “арккосинус”, или ​\( arccos(x) \)​.{-1}(x) \)​ часто используется для обозначения функции “арктангенс”, но его не рекомендуется употреблять, чтобы не путать с функцией ​\( 1/tg(x) \)​. — Примеч. ред.)

Срываем покров с векторов

У нас есть два способа описания векторов для решения физических задач. Первый основан на использовании компонент по осям X и Y, а второй — на величине (модуле) и направлении вектора (угол обычно задается в градусах от 0° до 360°, где угол 0° соответствует направлению вдоль положительного направления оси X). Знание правил взаимного преобразования этих двух способов описания имеет очень большое значение, поскольку для операций с векторами удобно использовать компоненты вектора, а в формулировке физических задач обычно задаются величины и углы векторов.

Вот как выглядит формула преобразования двух способов описания векторов:

В этом уравнении предполагается, что ​\( \theta \)​ — это угол между горизонтальной компонентой и гипотенузой ​\( h \)​ (т.е. самой длинной стороной прямоугольного треугольника, расположенного напротив прямого угла), как показано на рис. 4.8. Если угол не известен, то его можно вывести, если запомнить, что сумма всех углов треугольника равна 180°, а в прямоугольном треугольнике, если вычесть величину прямого угла 90°, то сумма остальных двух углов равна 90°.

Если вам известны компоненты (х,у), то его величину и направление можно определить по следующим формулам:

Такого рода преобразования нужно уметь легко выполнять, поскольку они довольно часто встречаются в задачах. На этом месте часто многие приходят в растерянность и не могут освоить дальнейший материал именно потому, что не овладели простыми правилами разложения вектора на компоненты.

Перемещение — тоже вектор

Перемещение ​\( s \)​ следует обозначать ​\( \mathbf{s} \)​, как вектор с определенной величиной и направлением (для обозначения векторов иногда используют стрелку, которая располагается над именем переменной, например ​\( \overrightarrow{s} \)​ ). В реальном мире очень важно знать не только величину, но и направление перемещения.

Допустим, что сбылись ваши детские мечты и вы стали звездой бейсбола. Вот вам нужно стремглав бежать к первой базе на расстоянии 90 футов по прямой. Но в каком направлении находится первая база? Допустим, что она находится под углом 45°, как показано на рис. 4.9. Тогда вектор вашего перемещения \( \mathbf{s} \) имеет величину 90 футов и направление 45°. А какими будут компоненты этого вектора? Это очень просто:

Скорость — еще один вектор

Представьте себе, что вы бежите к первой базе с вектором перемещения s с величиной 90 футов и направлением 45° по отношению к оси X. Тут стоило бы задаться вопросом: “Позволит мне моя скорость опередить игрока на первой базе?” Хороший вопрос. Достанем калькулятор и подсчитаем скорость, если известно, что для достижения первой базы вам требуется 3 секунды. Для определения скорости нужно поделить величину вектора \( \mathbf{s} \) на это время:

В этом выражении вектор перемещения делится на скаляр времени. Результатом такого деления является тоже вектор, а именно вектор скорости:

Итак, ваша скорость равна 30 футам в секунду под углом 45° и эта скорость является вектором \( \mathbf{v} \). Деление вектора на скаляр дает вектор другой величины, но такого же направления. В данном примере деление вектора перемещения \( \mathbf{s} \) на скаляр времени дает в результате вектор \( \mathbf{v} \). Он имеет такую же величину, что и величина перемещения, деленная на величину времени, но теперь вектор \( \mathbf{v} \) также имеет определенное направление, которое определяется направлением вектора перемещения \( \mathbf{s} \). Итак, в данном примере мы научились манипулировать с векторами, как со скалярами в главе 3, и получать вектор в результате этих манипуляций.

Допустим, что после этих вычислений вы пришли к выводу, что такой скорости недостаточно, чтобы опередить соперника. Ну что ж, нужно срочно изменить направление!

Ускорение — еще один вектор

Что произойдет, если в процессе движения внезапно изменить направление? Вы сразу же почувствуете изменение скорости, а значит, ощутите ускорение. Как и скорость, ускорение \( \mathbf{a} \) является вектором.

Предположим, что в предыдущем примере нужно изменить скорость Y-компоненты скорости до величины 25 футов в секунду, чтобы избежать встречи с соперником, причем вам известно, что вы способны отклониться от курса на 90° с ускорением 60 футов в секунду в квадрате (в отчаянной попытке увильнуть от соперника). Достаточно ли этого ускорения для изменения скорости за ту долю секунды, которая отделяет вас от встречи с соперником?

Разница конечного ​\( t_1 \)​ и начального ​\( t_0 \)​ момента времени равняется изменению времени ​\( \Delta t \)​. Теперь изменение скорости легко найти по следующей формуле:

Теперь попробуем вычислить изменение скорости от исходной скорости на основе данных на рис. 4.10.

Для поиска конечного значения скорости \( \mathbf{v_1} \) нужно выполнить операцию сложения векторов. Это значит, что нужно разложить на компоненты вектор исходной скорости ​\( \mathbf{v_0} \)​ и вектор изменения скорости ​\( \Delta v \)​. Вот как выглядят компоненты исходной скорости ​\( \mathbf{v_0} \)​:

Полпути пройдено. Итак, чему равно изменение скорости ​\( \Delta\!\mathbf{v} \)​? Известно, что ​\( \Delta\!\mathbf{v}=\mathbf{a}\Delta\mathbf{t} \)​, а ​\( \mathbf{a} \)​ = 60 футов в секунду2 под углом 90° к прежнему направлению движения, как показано на рис. 4.10. Итак, подсчитаем величину изменения скорости ​\( \Delta\!\mathbf{v} \)​ по формуле ​\( \Delta\!\mathbf{v}=\mathbf{a}\Delta\mathbf{t} \)​:

Но что можно сказать о направлении \( \Delta\!\mathbf{v} \)? Если взглянуть на рис. 4.10, то можно увидеть, что изменение скорости \( \Delta\!\mathbf{v} \) направлено под углом 90° к текущему направлению движения, которое ориентировано под углом 45° к положительному направлению оси X. Следовательно, изменение скорости \( \Delta\!\mathbf{v} \) направлено под углом 135° к положительному направлению оси X. Теперь можно получить выражение для компонент вектора изменения скорости \( \Delta\!\mathbf{v} \):

Теперь остается только выполнить сложение векторов для поиска конечной скорости:

Итак, получен результат ​\( \mathbf{v_1} \)​ = (17,0 фута в секунду; 25,4 фута в секунду). Y-компонента конечной скорости больше необходимой величины, которая равна 25,0 фута в секунду. После завершения этих вычислений можно спрятать калькулятор и смело выполнить запланированный вираж. Представьте себе, что к изумлению окружающих вам удалось уклониться от соперника и успешно достигнуть места назначения — первой базы (какой крутой поворот вам пришлось для этого выполнить!). Болельщики изумлены и приветствуют вас, а вы, небрежно касаясь кепки кончиками пальцев, отдаете им честь, зная, что все это стало возможным благодаря превосходному знанию физики. После затишья вы украдкой бросаете взгляд на вторую базу: а не закрепить ли успех и попробовать добежать до второй базы? Для этого снова придется достать калькулятор и определить компоненты векторов.

Именно так нужно работать с векторами разных физических параметров: перемещения, скорости и ускорения. Теперь, обладая такими знаниями, можно перевести скалярные уравнения из главы 3 в векторную форму, например, вот так:

Обратите внимание, что полный вектор перемещения — это комбинация перемещения с начальной скоростью и перемещения с постоянным ускорением.

Упражнение со скоростью: скользим по радуге

Хотя сила гравитации подробно описывается в главе 6, но здесь мы рассмотрим результат действия этой силы на небольшом примере с векторами в двух измерениях. Представьте себе, что мячик для игры в гольф движется по горизонтальной вершине скалы со скоростью 1,0 м/с и вскоре сорвется с края скалы на высоте 5 метров от поверхности Земли, как показано на рис. 4.11. Насколько далеко улетит мячик и с какой скоростью он столкнется с поверхностью Земли? В этой задаче прежде всего нужно определить время движения мячика.

Приступим к сбору фактов. Нам известно, что компоненты скорости мячика равны (1; 0), и он находится на высоте 5 метров от поверхности Земли. В процессе падения под действием силы тяготения Земли он движется с постоянным ускорением, ​\( g \)​, величина которого равна около 9,8 м/с2.

Итак, как определить, насколько далеко он упадет от края скалы? Один из способов решения этой задачи основан на определении времени движения мячика до столкновения с поверхностью Земли. Поскольку мячик ускоряется только в направлении оси Y (т.е. вертикально вниз), а его компонента скорости по оси X, ​\( v_x \)​, не меняется, то пройденное по горизонтали расстояние до столкновения будет равно ​\( v_xt \)​, где ​\( t \)​ — время движения мячика до столкновения. Сила тяготения ускоряет мячик по вертикали, а значит, перемещение по вертикали (т.е. вдоль оси Y) равно:

В данном случае перемещение по вертикали ​\( s_y \)​ = 5 метров, а ускорение ​\( a_y \)​ = ​\( g \)​ = 9,8 м/с2. Поэтому предыдущее уравнение принимает вид:

Это значит, что время движения мячика до столкновения равно:

Итак, мы вычислили, что мячик будет находиться в полете 1,0 секунды. Отлично, явный прогресс! Поскольку компонента скорости мячика по оси X не изменялась в течение этого времени, то можно легко вычислить расстояние, которое пролетит мячик по горизонтали (т.е. вдоль оси X) за это время:

Подставляем числа и получаем:

Итак, мячик столкнется с поверхностью Земли на расстоянии 1,0 метра по горизонтали.

Теперь можно приступать ко второму вопросу задачи: попробуем определить скорость мячика в момент столкновения с поверхностью Земли. Частично ответ на этот вопрос мы уже знаем, поскольку компонента скорости мячика по оси X не изменялась. Однако по вертикали сила тяготения ускорила мячик по вертикали (т.е. вдоль оси Y), а потому компоненты конечной скорости имеют следующий вид: (1,0; ?). Итак, нам нужно определить величину компоненты скорости мячика по оси Y, обозначенной вопросительным знаком. Воспользуемся следующим соотношением для компоненты скорости по вертикали:

В данном случае начальная скорость ​\( v_{y0} \)​ = 0, постоянное ускорение ​\( a_y=g \)​ и нужно определить только конечную скорость \( v_{y1} \). Поэтому предыдущее уравнение приобретает следующий вид:

Подставляем числа и получаем:

Ускорение свободного падения, ​\( g \)​, также является вектором ​\( \mathbf{g} \)​. Он направлен к центру Земли, т.е. в отрицательном направлении оси Y, а на поверхности Земли его величина равна около -9,8 м/с2.

Отрицательный знак здесь обозначает направление вниз вектора \( \mathbf{g} \), т.е. в отрицательном направлении оси Y. Итак, подставляем обновленное значение ускорения и получаем:

Итак, компоненты конечной скорости мячика равны (1,0; -9,8) м/с. Чтобы найти величину вектора скорости (а не его отдельных компонент) в момент столкновения с поверхностью Земли, выполним следующие вычисления:

Триумфальный финал! Мячик пролетит 1,0 метра по горизонтали и столкнется с поверхностью Земли со скоростью 9,9 м/с. Совсем неплохо для начала.

Глава 4. Едем по указателям

5 (100%) 2 votes

векторов | Безграничная физика

Компоненты вектора

Векторы — это геометрические представления величины и направления, которые могут быть выражены в виде стрелок в двух или трех измерениях.

Цели обучения

Контрастность двумерных и трехмерных векторов

Основные выводы

Ключевые моменты
  • Векторы можно разбить на две составляющие: величину и направление.
  • Взяв вектор, который нужно проанализировать, как гипотенузу, можно найти горизонтальную и вертикальную составляющие, заполнив прямоугольный треугольник.Нижний край треугольника — это горизонтальная составляющая, а сторона, противоположная углу, — вертикальная составляющая.
  • Угол, который вектор образует с горизонталью, можно использовать для вычисления длины двух компонентов.
Ключевые термины
  • координаты : числа, указывающие положение относительно некоторой оси. Пример: [latex] \ text {x} [/ latex] и [latex] \ text {y} [/ latex] координаты указывают положение относительно [latex] \ text {x} [/ latex] и [latex] \ text {y} [/ latex] топоры.
  • ось : воображаемая линия, вокруг которой объект вращается или симметрично расположен.
  • величина : Число, присвоенное вектору, указывающее его длину.

Обзор

Векторы — это геометрические представления величины и направления, которые часто представлены прямыми стрелками, начинающимися в одной точке на координатной оси и заканчивающимися в другой точке. Все векторы имеют длину, называемую величиной, которая представляет некоторое интересное качество, так что вектор можно сравнивать с другим вектором.Векторы, будучи стрелками, тоже имеют направление. Это отличает их от скаляров, которые представляют собой простые числа без направления.

Вектор определяется своей величиной и ориентацией относительно набора координат. При анализе векторов часто бывает полезно разбить их на составные части. Для двумерных векторов эти компоненты бывают горизонтальными и вертикальными. Для трехмерных векторов компонент величины такой же, но компонент направления выражается в терминах [латекс] \ text {x} [/ latex], [latex] \ text {y} [/ latex] и [latex] \ text {z} [/ латекс].

Разложение вектора

Чтобы визуализировать процесс разложения вектора на его компоненты, начните с рисования вектора из начала набора координат. Затем нарисуйте прямую линию от начала координат по оси x до тех пор, пока линия не сравняется с концом исходного вектора. Это горизонтальная составляющая вектора. Чтобы найти вертикальный компонент, нарисуйте линию прямо вверх от конца горизонтального вектора, пока не дойдете до конца исходного вектора. Вы должны обнаружить, что у вас есть прямоугольный треугольник, в котором исходный вектор является гипотенузой.

Разложение вектора на горизонтальные и вертикальные компоненты — очень полезный метод для понимания физических задач. Всякий раз, когда вы видите движение под углом, вы должны думать о нем как о движении одновременно по горизонтали и вертикали. Такое упрощение векторов может ускорить вычисления и помочь отслеживать движение объектов.

Скаляры и векторы : Г-н Андерсен объясняет различия между скалярными и векторными величинами.Он также использует демонстрацию, чтобы показать важность векторов и сложения векторов.

Компоненты вектора : исходный вектор, определенный относительно набора осей. Горизонтальный компонент простирается от начала вектора до его самой дальней координаты x. Вертикальный компонент простирается от оси x до самой вертикальной точки вектора. Вместе два компонента и вектор образуют прямоугольный треугольник.

Скаляры против векторов

Скаляры — это физические величины, представленные одним числом, а векторы представлены как числом, так и направлением.

Цели обучения

Определите разницу между скалярами величин и векторами, которые представляют

Основные выводы

Ключевые моменты
  • Скаляры — это физические величины, представленные одним числом без направления.
  • Векторы — это физические величины, требующие как величины, так и направления.
  • Примеры скаляров включают высоту, массу, площадь и объем. Примеры векторов включают смещение, скорость и ускорение.
Ключевые термины
  • Оси координат : набор перпендикулярных линий, определяющих координаты относительно начала координат. Пример: оси координат x и y определяют горизонтальное и вертикальное положение.

Физические величины обычно можно разделить на две категории: векторы и скаляры. Эти две категории типичны в зависимости от того, какая информация им требуется. Векторы требуют двух частей информации: величины и направления. Напротив, скаляры требуют только величины.Скаляры можно рассматривать как числа, тогда как векторы следует рассматривать как стрелки, указывающие в определенном направлении.

Вектор : пример вектора. Векторы обычно представлены стрелками, длина которых представляет величину, а направление — направлением, указанным стрелкой.

Векторы требуют как величины, так и направления. Величина вектора — это число для сравнения одного вектора с другим. В геометрической интерпретации вектора вектор представлен стрелкой.Стрелка состоит из двух частей, определяющих ее. Две части — это его длина, которая представляет величину и направление относительно некоторого набора осей координат. Чем больше величина, тем длиннее стрелка. Физические понятия, такие как смещение, скорость и ускорение, являются примерами величин, которые могут быть представлены векторами. Каждая из этих величин имеет как величину (как далеко или как быстро), так и направление. Чтобы указать направление, должно быть что-то, относительно чего это направление.Обычно эта контрольная точка представляет собой набор осей координат, таких как плоскость x-y.

Скаляры отличаются от векторов тем, что у них нет направления. Скаляры используются в основном для представления физических величин, для которых направление не имеет смысла. Вот некоторые из них: масса, высота, длина, объем и площадь. Говорить о направлении этих величин не имеет смысла, и поэтому они не могут быть выражены в виде векторов.

Разница между векторами и скалярами, Введение и основы : В этом видео представлена ​​разница между скалярами и векторами.Представлены идеи о величине и направлении, а также приведены примеры векторов и скаляров.

Сложение и вычитание векторов графически

Векторов можно добавлять или вычитать графически, накладывая их встык по набору осей.

Цели обучения

Смоделируйте графический метод сложения и вычитания векторов

Основные выводы

Ключевые моменты
  • Чтобы добавить векторы, положите первую на наборе осей хвостом в начале координат.Поместите следующий вектор хвостом в голову предыдущего вектора. Когда векторов больше нет, проведите прямую линию от начала до конца последнего вектора. Эта линия представляет собой сумму векторов.
  • Чтобы вычесть векторы, действуйте так, как если бы складывались два вектора, но переверните вектор для вычитания по осям, а затем соедините его хвостом к голове, как если бы складывались.
  • Сложение или вычитание любого количества векторов дает результирующий вектор.
Ключевые термины
  • начало координат : центр координатной оси, определенный как координата 0 по всем осям.
  • Оси координат : набор перпендикулярных линий, определяющих координаты относительно начала координат. Пример: оси координат x и y определяют горизонтальное и вертикальное положение.

Сложение и вычитание векторов

Одним из способов, которым представление физических величин в виде векторов упрощает анализ, является легкость, с которой векторы могут быть добавлены друг к другу. Поскольку векторы представляют собой графические визуализации, сложение и вычитание векторов можно выполнять графически.

Графический метод сложения векторов также известен как метод «голова к хвосту». Для начала нарисуйте набор осей координат. Затем нарисуйте первый вектор с его хвостом (основанием) в начале координат осей. Для сложения векторов не имеет значения, какой вектор вы рисуете первым, поскольку сложение коммутативно, но для вычитания убедитесь, что вектор, который вы рисуете первым, это тот, который вы вычитаете из . Следующий шаг — взять следующий вектор и нарисовать его так, чтобы его хвост начинался с головы предыдущего вектора (сторона стрелки).Продолжайте помещать каждый вектор в начало предыдущего, пока все векторы, которые вы хотите добавить, не будут объединены. Наконец, проведите прямую линию от начала координат до головы последнего вектора в цепочке. Эта новая линия является векторным результатом сложения этих векторов вместе.

Графическое сложение векторов : Метод сложения векторов «голова к хвосту» требует, чтобы вы расположили первый вектор вдоль набора осей координат. Затем поместите хвост следующего вектора на голову первого.Нарисуйте новый вектор от начала до конца последнего вектора. Этот новый вектор представляет собой сумму двух исходных.

Сложение векторов Урок 1 из 2: Метод сложения «голова к хвосту» : Это видео знакомит зрителей с добавлением и вычитанием векторов. В первом уроке показано графическое сложение, а во втором видео используется более математический подход и показано сложение векторов по компонентам.

Метод вычитания векторов аналогичен.Убедитесь, что первый вектор, который вы рисуете, — это тот, из которого нужно вычесть. Затем, чтобы вычесть вектор, действуйте так, как если бы добавляли напротив этого вектора. Другими словами, переверните вектор, который нужно вычесть, по осям, а затем соедините его хвостом к голове, как будто складывая. Чтобы перевернуть вектор, просто поместите его голову на место хвоста, а хвост на место головы.

Сложение и вычитание векторов с использованием компонентов

Часто проще складывать или вычитать векторы, используя их компоненты.

Цели обучения

Продемонстрируйте, как складывать и вычитать векторы по компонентам

Основные выводы

Ключевые моменты
  • Векторы можно разложить на горизонтальные и вертикальные компоненты.
  • После того, как векторы разложены на компоненты, можно добавлять компоненты.
  • Сложение соответствующих компонентов двух векторов дает вектор, который является суммой двух векторов.
Ключевые термины
  • Компонент : часть вектора.Например, горизонтальная и вертикальная составляющие.

Использование компонентов для сложения и вычитания векторов

Другой способ добавления векторов — это добавление компонентов. Ранее мы видели, что векторы можно выразить через их горизонтальные и вертикальные компоненты. Чтобы добавить векторы, просто выразите их оба в терминах их горизонтальных и вертикальных компонентов, а затем сложите компоненты вместе.

Вектор с горизонтальными и вертикальными компонентами : вектор на этом изображении имеет величину 10.3 единицы и направление на 29,1 градуса выше оси абсцисс. Его можно разделить на горизонтальную и вертикальную части, как показано на рисунке.

Например, вектор длиной 5 под углом 36,9 градуса к горизонтальной оси будет иметь горизонтальную составляющую 4 единицы и вертикальную составляющую 3 единицы. Если бы мы добавили это к другому вектору той же величины и направления, мы бы получили бы вектор вдвое большей длины под тем же углом. Это можно увидеть, сложив горизонтальные компоненты двух векторов ([latex] 4 + 4 [/ latex]) и двух вертикальных компонентов ([latex] 3 + 3 [/ latex]).Эти добавления дают новый вектор с горизонтальной составляющей 8 ([latex] 4 + 4 [/ latex]) и вертикальной составляющей 6 ([latex] 3 + 3 [/ latex]). Чтобы найти результирующий вектор, просто поместите хвост вертикального компонента в головку (сторона стрелки) горизонтального компонента, а затем проведите линию от начала до вершины вертикального компонента. Эта новая строка является результирующим вектором. Он должен быть вдвое длиннее оригинала, так как оба его компонента в два раза больше, чем были ранее.

Чтобы вычесть векторы по компонентам, просто вычтите два горизонтальных компонента друг из друга и сделайте то же самое для вертикальных компонентов. Затем нарисуйте получившийся вектор, как вы делали в предыдущей части.

Добавление векторов Урок 2 из 2: Как добавлять векторы по компонентам : Это видео знакомит зрителей с добавлением векторов с использованием математического подхода и демонстрирует сложение векторов по компонентам.

Умножение векторов на скаляр

Умножение вектора на скаляр изменяет величину вектора, но не направление.

Цели обучения

Обобщить взаимодействие между векторами и скалярами

Основные выводы

Ключевые моменты
  • Вектор — это величина, имеющая как величину, так и направление.
  • Скаляр — это величина, имеющая только величину.
  • Умножение вектора на скаляр эквивалентно умножению величины вектора на скаляр. Вектор удлиняется или сжимается, но не меняет направления.
Ключевые термины
  • вектор : Направленная величина, имеющая как величину, так и направление; между двумя точками.
  • величина : Число, присвоенное вектору, указывающее его длину.
  • скаляр : величина, имеющая величину, но не направление; сравнить вектор.

Обзор

Хотя векторы и скаляры представляют различные типы физических величин, иногда необходимо, чтобы они взаимодействовали. Хотя добавление скаляра к вектору невозможно из-за их различных пространственных размеров, вектор можно умножить на скаляр.Однако скаляр нельзя умножить на вектор.

Чтобы умножить вектор на скаляр, просто умножьте аналогичные компоненты, то есть величину вектора, на величину скаляра. Это приведет к новому вектору с тем же направлением, но произведению двух величин.

Пример

Например, если у вас есть вектор A с определенной величиной и направлением, умножение его на скаляр a с величиной 0,5 даст новый вектор с величиной, равной половине исходной.Точно так же, если вы возьмете число 3, которое является чистым скаляром без единиц измерения, и умножите его на вектор, вы получите версию исходного вектора, которая в 3 раза длиннее. В качестве более физического примера возьмем гравитационную силу, действующую на объект. Сила — это вектор, величина которого зависит от скаляра, известного как масса, и его направления вниз. Если масса объекта удваивается, сила тяжести также удваивается.

Умножение векторов на скаляры очень полезно в физике. Большинство единиц, используемых в векторных величинах, по своей сути являются скалярами, умноженными на вектор.Например, единица измерения скорости в метрах в секунду, которая является вектором, состоит из двух скаляров, которые являются величинами: скаляр длины в метрах и скаляр времени в секундах. Чтобы преобразовать величины в скорость, нужно умножить единичный вектор в определенном направлении на эти скаляры.

Скалярное умножение : (i) Умножение вектора [latex] \ text {A} [/ latex] на скаляр [latex] \ text {a} = 0,5 [/ latex] дает вектор [latex] \ text { B} [/ latex] который вдвое короче.(ii) Умножение вектора [латекс] \ text {A} [/ latex] на 3 увеличивает его длину в три раза. (iii) Удвоение массы (скаляр) удваивает силу (вектор) гравитации.

Единичные векторы и умножение на скаляр

Умножение вектора на скаляр — это то же самое, что умножение его величины на число.

Цели обучения

Предсказать влияние умножения вектора на скаляр

Основные выводы

Ключевые моменты
  • Единичный вектор — это вектор величины (длины) 1.
  • Скаляр — это физическая величина, которая может быть представлена ​​одним числом. В отличие от векторов, скаляры не имеют направления.
  • Умножение вектора на скаляр — это то же самое, что умножение величины вектора на число, представленное скаляром.
Ключевые термины
  • скаляр : Величина, которая может быть описана одним числом, в отличие от вектора, который требует направления и числа.
  • единичный вектор : вектор величины 1.

Помимо сложения векторов, векторы также можно умножать на константы, известные как скаляры. Скаляры отличаются от векторов тем, что они представлены величиной, но не направлением. Примеры скаляров включают массу, высоту или объем объекта.

Скалярное умножение : (i) Умножение вектора A на 0,5 уменьшает его длину вдвое. (ii) Умножение вектора A на 3 увеличивает его длину втрое. (iii) Увеличение массы (скаляр) увеличивает силу (вектор).

При умножении вектора на скаляр направление вектора не изменяется, а величина умножается на величину скаляра. Это приводит к тому, что новая векторная стрелка указывает в том же направлении, что и старая, но с большей или меньшей длиной. Вы также можете выполнить скалярное умножение с помощью компонентов вектора. Когда у вас есть компоненты вектора, умножьте каждый из компонентов на скаляр, чтобы получить новые компоненты и, следовательно, новый вектор.

Полезной концепцией при изучении векторов и геометрии является концепция единичного вектора. Единичный вектор — это вектор с длиной или величиной, равной единице. Единичные векторы различны для разных координат. В декартовых координатах направлениями являются x и y, обычно обозначаемые [latex] \ hat {\ text {x}} [/ latex] и [latex] \ hat {\ text {y}} [/ latex]. С треугольником над буквами называется «шляпа». Единичные векторы в декартовых координатах описывают круг, известный как «единичный круг» с радиусом один.Это можно увидеть, взяв все возможные векторы длины один под всеми возможными углами в этой системе координат и поместив их в координаты. Если бы вы провели линию, соединяющую все головы всех векторов вместе, вы бы получили круг радиуса один.

Положение, смещение, скорость и ускорение как векторы

Положение, смещение, скорость и ускорение могут быть показаны векторами, поскольку они определены в терминах величины и направления.

Цели обучения

Изучить применение векторов в анализе физических величин

Основные выводы

Ключевые моменты
  • Векторы — это стрелки, состоящие из величины и направления. Они используются в физике для обозначения физических величин, которые также имеют как величину, так и направление.
  • Смещение — это физический термин, означающий расстояние объекта от контрольной точки. Поскольку смещение содержит две части информации: расстояние от опорной точки и направление от точки, оно хорошо представлено вектором.
  • Скорость определяется как скорость изменения смещения во времени. Чтобы узнать скорость объекта, нужно знать, как быстро изменяется смещение, и в каком направлении. Следовательно, он также хорошо представлен вектором.
  • Ускорение, являющееся скоростью изменения скорости, также требует как величины, так и направления относительно некоторых координат.
  • При рисовании векторов часто не хватает места, чтобы нарисовать их в масштабе, который они представляют, поэтому важно где-нибудь указать, в каком масштабе они нарисованы.
Ключевые термины
  • скорость : Скорость изменения смещения относительно изменения во времени.
  • смещение : длина и направление прямой линии между двумя объектами.
  • ускорение : скорость, с которой скорость тела изменяется со временем

Использование векторов

Векторы могут использоваться для представления физических величин. Чаще всего в физике векторы используются для обозначения смещения, скорости и ускорения.Векторы представляют собой комбинацию величины и направления и отображаются в виде стрелок. Длина представляет собой величину, а направление этой величины — это направление, в котором указывает вектор. Поскольку векторы строятся таким образом, полезно анализировать физические величины (как с размером, так и с направлением) как векторами.

Приложения

В физике векторы полезны, потому что они могут визуально представлять положение, смещение, скорость и ускорение. При рисовании векторов у вас часто не хватает места, чтобы нарисовать их в масштабе, который они представляют, поэтому важно где-то обозначить, в каком масштабе они нарисованы.Например, при рисовании вектора, представляющего величину 100, можно нарисовать линию длиной 5 единиц в масштабе [латекс] \ displaystyle \ frac {1} {20} [/ latex]. Когда величина, обратная шкале, умножается на нарисованную величину, она должна равняться действительной величине.

Положение и перемещение

Смещение определяется как расстояние объекта в любом направлении относительно положения другого объекта. Физики используют концепцию вектора положения как графический инструмент для визуализации смещений.Вектор положения выражает положение объекта от начала системы координат. Вектор положения также можно использовать для отображения положения объекта относительно опорной точки, вторичного объекта или исходного положения (при анализе того, насколько далеко объект переместился от своего исходного положения). Вектор положения — это прямая линия, проведенная от произвольной исходной точки к объекту. После рисования вектор имеет длину и направление относительно используемой системы координат.

Скорость

Скорость также определяется величиной и направлением.Чтобы сказать, что что-то набирает или теряет скорость, нужно также сказать, насколько и в каком направлении. Например, самолет, летящий в 200 [latex] \ frac {\ text {km}} {\ text {h}} [/ latex] на северо-восток, может быть представлен вектором, указывающим в северо-восточном направлении, с магнитудой 200 [латекс] \ frac {\ text {km}} {\ text {h}} [/ latex]. При рисовании вектора величина важна только как способ сравнения двух векторов одинаковых единиц. Итак, если бы другой самолет летел на 100 [latex] \ frac {\ text {km}} {\ text {h}} [/ latex] на юго-запад, векторная стрелка должна быть вдвое короче и указывать в направлении юго-запад.

Разгон

Ускорение, представляющее собой скорость изменения скорости во времени, складывается из величины и направления и строится по той же концепции, что и вектор скорости. Значение ускорения не помогло бы в физике, если бы величина и направление этого ускорения были неизвестны, поэтому эти векторы важны. На диаграмме свободного тела, например, падающего объекта, было бы полезно использовать вектор ускорения рядом с объектом, чтобы обозначить его ускорение по направлению к земле.2} [/ латекс] .

Векторная диаграмма : Мужчина поднимается на холм. Его направление движения определяется углом тета относительно вертикальной оси и длиной стрелки, идущей вверх по холму. Он также ускоряется вниз под действием силы тяжести.

Как складывать и вычитать векторы (с диаграммами)

Обновлено 28 декабря 2020 г.

Автор GAYLE TOWELL

Вектор — это величина, с которой связаны как величина, так и направление.Это отличается от скалярной величины , которая соответствует только величине. Скорость — это пример векторной величины. У него есть и величина (как быстро что-то движется), и направление (направление, в котором оно движется).

Векторы часто изображаются в виде стрелок. Длина стрелки соответствует величине вектора, а острие стрелки указывает направление.

Есть два способа работы с векторным сложением и вычитанием. Первый — графически, манипулируя стрелочными диаграммами самих векторов.Второй — математический, который дает точные результаты.

Сложение и вычитание графических векторов в одном измерении

При сложении двух векторов вы помещаете хвост второго вектора на вершину первого вектора, сохраняя ориентацию вектора. Результирующий вектор — это вектор, который начинается в хвосте первого вектора и указывает по прямой линии до конца второго вектора.

Например, рассмотрите возможность сложения векторов A и B , которые указывают в одном направлении вдоль линии.Мы размещаем их «кончик к хвосту», и результирующий вектор C указывает в том же направлении и имеет длину, которая является суммой длин A и B .

Вычитание векторов в одном измерении по сути то же самое, что и сложение, за исключением того, что вы «переворачиваете» второй вектор. Это напрямую связано с тем, что вычитание — это то же самое, что и добавление отрицательного числа.

Математическое сложение и вычитание векторов в одном измерении

При работе в одном измерении направление вектора может быть указано знаком.Мы выбираем одно направление в качестве положительного (обычно «вверх» или «вправо» выбираются как положительные) и назначаем любой вектор, указывающий в этом направлении, как положительную величину. Любой вектор, указывающий в отрицательном направлении, является отрицательной величиной. При сложении или вычитании векторов складывайте или вычитайте их величины с соответствующими знаками.

Предположим, что в предыдущем разделе вектор A имел величину 3, а вектор B имел величину 5. Тогда результирующий вектор C = A + B = 8, вектор величина 8 указывает в положительном направлении, и результирующий вектор D = A — B = -2, вектор величины 2 указывает в отрицательном направлении.Обратите внимание, что это согласуется с предыдущими графическими результатами.

Совет: будьте осторожны, складывайте только векторы одного и того же типа: скорость + скорость, сила + сила и т. Д. Как и во всей математике в физике, единицы должны совпадать!

Сложение и вычитание графических векторов в двух измерениях

Если первый вектор и второй вектор не находятся на одной линии в декартовом пространстве, вы можете использовать тот же метод «кончик к хвосту», чтобы сложить или вычесть их. Чтобы сложить два вектора, просто представьте, что вы поднимаете второй и кладете его хвост на кончик первого, сохраняя его ориентацию, как показано.Результирующий вектор представляет собой стрелку, начинающуюся в конце первого вектора и заканчивающуюся на вершине второго вектора:

Как и в одном измерении, вычитание одного вектора из другого эквивалентно переворачиванию и сложению. Графически это выглядит следующим образом:

••• Дана Чен | Sciencing

Примечание: Иногда сложение векторов отображается графически, складывая хвосты двух слагаемых векторов вместе и создавая параллелограмм. Результирующий вектор будет диагональю этого параллелограмма.

Математическое сложение и вычитание векторов в двух измерениях

Чтобы математически сложить и вычесть векторы в двух измерениях, выполните следующие действия:

    Разложите каждый вектор на компонент размером x , иногда называемый горизонтальным компонентом, и компонент y , иногда называемый вертикальным компонентом, с использованием тригонометрии. (Обратите внимание, что компоненты могут быть как отрицательными, так и положительными, в зависимости от того, в каком направлении указывает вектор)

    Сложите x -компоненты обоих векторов вместе, а затем сложите y -компоненты обоих векторов вместе .Этот результат дает вам компоненты x и y результирующего вектора.

    Величину результирующего вектора можно найти с помощью теоремы Пифагора.

    Направление результирующего вектора можно найти с помощью тригонометрии, используя функцию обратного тангенса. Это направление обычно задается как угол по отношению к положительной оси x .

Тригонометрия в сложении векторов

Вспомните отношения между сторонами и углами прямоугольного треугольника из тригонометрии.2

Движение снаряда представляет собой классические примеры того, как мы можем использовать эти отношения как для разложения вектора, так и для определения окончательной величины и направления вектора.

Представьте, что два человека играют в мяч. Предположим, вам говорят, что мяч брошен с высоты 1,3 м со скоростью 16 м / с под углом 50 градусов к горизонту. Чтобы начать анализ этой проблемы, вам необходимо разложить этот вектор начальной скорости на компоненты x и y , как показано:

v_ {xi} = v_i \ cos (\ theta) = 16 \ раз \ cos (50) = 10.{-1} \ Big (\ frac {-13.3} {10.3} \ Big) = — 52,2 \ градус

Пример сложения и вычитания векторов

Представьте, что автомобиль скручивает угол. Предположим, что v i для автомобиля находится в направлении x- с величиной 10 м / с, а v f находится под углом 45 градусов к положительному значению . x — ось с магнитудой 10 м / с. Если это изменение движения происходит через 3 секунды, какова величина и направление ускорения автомобиля при повороте?

Напомним, что ускорение a — это векторная величина, определяемая как:

a = \ frac {(v_f-v_i)} {t}

Где v f и v i — конечная и начальная скорости соответственно (и, следовательно, также векторные величины).

Чтобы вычислить разность векторов v f v i , , мы должны сначала разложить начальный и конечный векторы скорости:

v_ {xi} = 10 \ text {m / s} \\ v_ {yi} = 0 \ text {m / s} \\ v_ {xf} = 10 \ cos (45) = 7.07 \ text {m / s} \\ v_ {yf} = 10 \ sin (45) = 7.07 \ text {m / s}

Затем мы вычитаем окончательные компоненты x и y из исходных компонентов x и y . чтобы получить компоненты v f v i :

Затем мы вычитаем компоненты x и y :

(v_f- v_i) _x = v_ {xf} -v_ {xi} = 7.{-1} \ Big (\ frac {2.36} {- 0.977} \ Big) = 113 \ градус

Сложение векторов — объяснение и примеры

Подобно скалярному сложению, сложение векторов включает объединение двух или более векторов. Более конкретно, когда вы складываете векторы, вы:

«Добавление двух или более векторов с помощью операции сложения для получения нового вектора, равного сумме двух или более векторов».

В этом разделе мы обсудим сложение векторов со следующих аспектов:

  • Что такое сложение векторов?
  • Как добавить векторы графически
  • Как добавить два вектора

Что такое сложение векторов?

Два вектора, A и B , могут быть сложены вместе с использованием векторного сложения, и результирующий вектор может быть записан как:

R = A + B

Как добавить векторы графически

При использовании сложения векторов мы должны учитывать оба компонента вектора, а именно направление и величину.

Имейте в виду, что два вектора с одинаковой величиной и направлением можно складывать как скаляры.

В этом разделе мы рассмотрим графические и математические методы сложения векторов, в том числе:

  1. Сложение векторов с использованием правила «голова к хвосту»
  2. Сложение векторов с использованием метода параллелограмма
  3. Сложение векторов с использованием компонентов

Сложение векторов с использованием правила «голова к хвосту»

Сложение векторов может быть выполнено с использованием известного метода «голова к хвосту».Согласно этому правилу, два вектора можно сложить вместе, поместив их вместе так, чтобы голова первого вектора соединялась с хвостом второго вектора. Результирующий вектор суммы затем может быть получен путем соединения хвоста первого вектора с заголовком второго вектора. Иногда это также называют методом сложения векторов треугольником.

Сложение векторов с использованием правила «голова к хвосту» показано на рисунке ниже. Два вектора P и Q складываются с использованием метода «голова к хвосту», и мы можем видеть треугольник, образованный двумя исходными векторами и вектором суммы.

Сначала два вектора P и Q помещаются вместе так, что голова вектора P соединяется с хвостом вектора Q . Затем, чтобы найти сумму, рисуется результирующий вектор R таким образом, чтобы он соединял хвост P с головкой Q .

Математически сумма или результирующий вектор R, на изображении ниже может быть выражена как:

R = P + Q

Сложение векторов с использованием метода параллелограмма

Чтобы понять сложения векторов методом параллелограмма мы рассмотрим и поясним рисунок ниже.

Сначала нарисуйте данные векторы, A, и B, , чтобы они имели ту же начальную точку, как показано на изображении ниже. Затем нарисуйте параллелограмм, используя копии данных векторов.

Во-вторых, нарисуйте копию вектора B , названную B ’, , и поместите ее параллельно вектору B, чтобы соединить с головкой первого вектора, A . Точно так же нарисуйте копию вектора A, , названную A ’, , и поместите ее параллельно A так, чтобы ее хвост соединялся с головкой вектора B.

Наконец, результат двух векторов, равный сумме векторов A, и B , будет диагональю параллелограмма. Его можно нарисовать, соединив начальную точку двух векторов A, и B с головкой векторов A ’ и B’ .

Таким образом, для выполнения сложения векторов с использованием метода параллелограмма требуется три шага:

Шаг 1: Поместите два вектора так, чтобы они имели общую начальную точку

Шаг 2: Нарисуйте и завершите параллелограмм, используя копии двух исходных векторов

Шаг 3: Тогда диагональ параллелограмма равна сумме двух векторов

Сложение векторов с использованием компонентов

Как мы знаем, векторы, заданные в декартовых координатах, можно разложить на их горизонтальные и вертикальные компоненты.Например, вектор P под углом Φ, как показано на изображении ниже, можно разложить на его компоненты следующим образом:

P x , который представляет компонент вектора P вдоль горизонтальной оси. (ось x) и

P y , который представляет компонент вектора P вдоль вертикальной оси (ось y).

Можно видеть, что три вектора образуют прямоугольный треугольник и что вектор P может быть выражен как:

P = P x + P y

Математически , компоненты вектора также могут быть вычислены с использованием величины и угла данного вектора.2

И угол может быть найден как:

Φ = tan -1 (P y / P x )

Таким образом, в итоге мы можем определить результирующий вектор, если его компоненты заданы. В качестве альтернативы, если задан сам вектор, мы можем определить компоненты, используя приведенные выше уравнения.

Точно так же, если векторы выражены в упорядоченных парах (векторах-столбцах), мы можем выполнить операцию сложения с векторами, используя их компоненты. Например, рассмотрим два вектора M и N , заданных как:

M = (m1, m2)

N = (n1, n2)

Выполнение векторного сложения двух векторов эквивалентно для добавления соответствующих компонентов x и y двух векторов.2

И угол можно вычислить как:

Φ = тангенс угла поворота -1 (S y / S x ).

Как сложить два вектора

В этом разделе обсуждаются примеры сложения векторов и их пошаговые решения, чтобы получить некоторую практику использования различных методов, описанных выше.

Примеры

Пример 1

Учитывая, что два вектора, A и B, как показано на изображении ниже, графически определяют свою сумму с использованием метода «голова к хвосту».

Решение

Первый шаг метода «голова к хвосту» — это размещение данных векторов A, и B таким образом, чтобы хвост вектора B соединялся с головой вектора A, , как показано на изображении ниже. Затем, чтобы найти их сумму, мы рисуем результирующий вектор R так, чтобы он соединял хвост вектора A с головкой вектора B . Математически результат может быть выражен как:

R = A + B

Пример 2

Учитывая два вектора, AB = (3, 2) и BC = ( 2, 2), определите их сумму по правилу «голова к хвосту».

Решение

AB + BC = (3, 2) + (2, 2)

AB + BC = (3 + 2, 2 + 2)

AB + BC = (5, 4).

Или, как показано на изображении ниже, результирующий вектор можно записать как:

AC = (5, 4)

Примечание. Чтобы использовать правило треугольника / правило «голова к хвосту», промежуточная буква два добавляемых вектора должны быть одинаковыми:

AC = AB + BC

В этом примере промежуточная буква B .2

| AC | = √ 25 + 16

| AC | = 6,403 единиц (приблизительно).

Угол результирующего вектора AC можно найти следующим образом:

Φ = tan -1 (AC y / AC x )

Φ = tan -1 (4/5 )

Φ = tan -1 (4/5)

Φ = 38,66 градусов

Пример 3

Для двух векторов S = 10 м, Φ = 30 градусов и T = 20 м , Φ = 60 градусов, определить их сумму.Затем вычислите величину и угол результирующего вектора, используя метод компонентов.

Решение

Пусть R будет результирующим вектором, равным сумме данных векторов, которая может быть выражена как:

R = S + T

Чтобы использовать компонентный метод , мы сначала смотрим на составные части данных векторов. Горизонтальная составляющая S составляет:

Sx = S Cos Φ

Sx = 10 Cos 30

Sx = 8.660 м (приблизительно)

Аналогично для вертикальной составляющей:

Sy = S Sin Φ

Sy = 10 Sin 30

Sy = 5 м

Затем мы вычисляем компоненты вектора T:

Tx = T Cos Φ

Ty = T Sin Φ

Где,

Tx = 20 Cos 60

Tx = 10 м

Ty = 20 Sin 60

Ty = 17.320 (приблизительно)

Теперь мы можем вычислить вектор суммы, сложив отдельные компоненты x и y вектора S и T следующим образом:

Rx = Sx + Tx

Rx = 8,660 + 10

Rx = 16,660 м

Ry = Sy + Ty

Ry = 5 + 17,32

Ry = 22,320 м

Результирующий вектор R можно представить в виде столбца как:

R = (16.2

| R | = 23,292 м (приблизительно)

Φ = tan -1 (Ry / Rx)

Φ = tan -1 (22,32 / 16,66)

Φ = 53,26 градуса (приблизительно)

Таким образом, результат вектор суммы:

R = 23,292 м, Φ = 53,26 градуса.

Пример 4

Путешественник идет P = 20 м прямо на запад, а затем Q = 10 м прямо на север. 2

| R | = 22.36 м (приблизительно)

И угол можно рассчитать как:

Φ = тангенциальный угол -1 (10/20)

Φ = 26,57 градуса

Таким образом, путешественник преодолел расстояние 22,36 м от стартовой позиции. точка под углом 26,57 градуса к северо-западу.

Пример 5

Определите результирующий вектор суммы для двух векторов A = (-5, -1) и B = (2, -1).

Решение

Указанные векторы уже находятся в своей компонентной форме, поэтому сначала мы определяем их углы.

Для вектора A:

Φ = tan -1 (Ay / Ax)

Φ = tan -1 (-1 / -5)

Φ = 11,31 градуса.

Для вектора B:

Φ = tan -1 (By / Bx)

Φ = tan -1 (-1/2)

Φ = -26,57 градусов.

Затем мы находим результирующий вектор, складывая отдельные компоненты:

S = A + B

Sx = Ax + Bx

Sx = -5 + 2

Sx = -3

Sy = Ay + By

Sy = -1-1

Sy = -2

Результирующий вектор S может быть выражен в виде столбца вектор:

S = (-3, -2).2

| S | = 3,605 единиц (приблизительно)

Φ = tan -1 (Sy / Sx)

Φ = tan -1 (-2 / -3)

Φ = 33,69 градусов

Таким образом, итоговая сумма вектор:

S = 3,605 единиц, Φ = 33,69 градуса.

Пример 6

Учитывая два вектора PQ и QR, , как показано на изображении ниже, вычислите значение их суммы, вектор PR .

Решение

Из данного изображения результирующий вектор может быть задан как:

PQ + QR = (2, 3) + (2, -2)

PQ + QR = (4, 1).2

| PR | = √ 17

| PR | = 4,123 единицы (приблизительно)

Угол результирующего вектора PR можно найти следующим образом:

Φ = tan -1 (1/4)

Φ = 14,04 градуса

Практические вопросы
  1. Для двух векторов, V = (2, 5) и C = (3, -2), определите их сумму, используя правило «голова к хвосту». Также определите величину и угол результирующего вектора R .
  2. Учитывая два вектора G = (5, 5) и H = (4, -10), определите их сумму, используя правило «голова-йо-хвост». Также определите величину и угол результирующего вектора P .
  3. Учитывая векторы OA, , где O = (-1, 3) и A = (5,2), и вектор UV, , где U = (1, -2) и V = (-2,2), определите результирующий вектор суммы S. Затем найдите его величину и угол.
  4. Учитывая четырехугольник ABCD, определите следующее:
  • DC + CA =?
  • BD + DC =?
  • AD + DC =?

  1. M = 10 м на восток и N = 15 м на север.Определите сумму двух векторов, затем найдите величину и угол результирующего вектора.

Ответы

  1. Результирующий вектор R равен R = (5, 3), величина R равна | R | = 5,830 единиц, а угол Φ = 30,96 градуса.
  2. Результирующий вектор P равен P = (9, 5), величина P равна | P | = 10. 30 единиц, а угол Φ = 29.05 градусов.
  3. Векторы: OA = (6, -1) и UV = (-3, 4), результирующий вектор суммы S задается как S = (3, 3), величина S есть | S | = 4,242 единицы, а угол Φ = 45 градусов.
  4. В данном четырехугольнике сумма вычисляется как:

DC + CA = DA

BD + DC = BC

AD + DC = AC

  1. Результат двух векторов:

R = M + N

| R | = 18.027 м,

И угол можно вычислить как:

Φ = тангенциальный угол -1 (15/10)

Φ = 56,30 градуса.

Таким образом, результирующий вектор равен R = 18,027 м , Φ = 56,30 градусов к северо-востоку.

Предыдущий урок | Главная страница | Следующий урок

Сложение векторов

Математика и естественные науки были изобретены людьми для описания и понять мир вокруг нас. Мы наблюдаем, что есть некоторые количества и процессы в наш мир, который зависит от направления , в котором они происходят, и есть некоторые количества, которые не зависят по направлению.Математики и ученые называют количество которое зависит от направления векторной величины . Количество которая не зависит от направления, называется скалярной величиной . А векторная величина имеет две характеристики: звездную величину и направление . Когда сравнение две векторные величины одного типа, необходимо сравнить обе величина и направление.

На этом слайде мы описываем метод добавления двух векторов.Сложение векторов — это один из аспектов большой векторной алгебры, которой мы являемся. , а не , будет присутствовать на этом сайте. Представлено сложение векторов здесь, потому что это происходит довольно часто при изучении движения и поскольку он демонстрирует некоторые фундаментальные различия между векторы и скаляры.

На рисунках векторы обычно обозначаются стрелкой. Длина стрелки указывает величину и кончик стрелки указывает направление.Вектор помечены алфавитным букву с чертой сверху, чтобы отличить ее от скаляра. Обозначим величину вектора символом | a | . Направление будет измеряться под углом фи относительно координаты ось х . Ось координат y перпендикулярна х . Примечание: Оси координат x и y сами по себе векторы! У них есть величина и направление.Сначала ты сталкиваются с осями координат, когда вы учитесь строить график. Так что у тебя есть какое-то время использовал векторы, даже не подозревая об этом!

Если мы построим пунктирную линию из кончика вектора , получится идущий параллельно оси x, он пересекает ось y в том месте, где мы этикетка ау . Аналогично линия от кончика вектора параллельно оси Y разрезает ось X на ax . Количества и и и называются составные части вектора, и оба являются скалярными величинами.

Чтобы сложить два вектора, a и b , сначала мы разбиваем каждый вектор на его компоненты, ax и ay , и bx и от , как показано на рисунке. Из правил, регулирующих равенство векторов синий вектор b равен черному вектору b потому что он имеет равную длину и одинаковое направление. Теперь, поскольку компоненты вектора a и вектор b — это скаляры, мы можем добавить x-компоненты для генерации x-компонент нового вектора c :

cx = ax + bx

Точно так же мы можем добавить y-компонентов :

cy = ay + по

Новые компоненты cx и cy полностью определяют новый вектор c , указав как величину, так и направление.Внимательно посмотрев на диаграмму, мы видим, что добавление двух векторов дает новый вектор, который равен , а не в направлении любого из исходные векторы, величина которых равна , а не , равной сумме величин исходных векторов. Векторная алгебра сильно отличается от скалярной алгебры, потому что она должна учитывать как величину, так и направление.

Примечание: На этом слайде для простоты мы разработали компоненты только в двух измерениях; есть две оси координат.На самом деле есть три пространственных измерения и три компонента все силы. Это важно при выводе общие уравнения с движением для траекторий полета и для Навье-Стокса и Уравнения Эйлера, которые описать силы и результирующее движение жидкостей в двигателе. Мы можем разбить очень сложные трехмерные векторные задачи на всего три скалярных уравнения.


Деятельность:

Экскурсии с гидом

Навигация..


Руководство для начинающих Домашняя страница

Как сложить или вычесть два вектора с помощью метода параллелограмма

Этапы сложения или вычитания двух векторов с помощью метода параллелограмма

Шаг 1: При добавлении двух векторов {eq} \ vec {v} + \ vec {w} {/ eq}, нарисуйте два вектора так, чтобы у них была одна и та же начальная точка. Если вычесть два вектора, {eq} \ vec {v} — \ vec {w} {/ eq}, сначала измените направление второго вектора, {eq} \ vec {w} {/ eq}, чтобы создать {eq} — \ vec {w} {/ экв}.Затем нарисуйте {eq} \ vec {v} \ text {and} — \ vec {w} {/ eq}, чтобы у них была одна и та же начальная точка.

Шаг 2: Нарисуйте параллелограмм с двумя векторами в качестве двух смежных сторон.

Шаг 3: Нарисуйте вектор от начальной точки двух векторов до противоположной вершины параллелограмма, чтобы создать диагональ параллелограмма. Этот вектор представляет собой сумму или разность двух векторов.

Словарь для сложения или вычитания двух векторов с помощью метода параллелограмма

Метод параллелограмма для сложения векторов: Если {eq} \ vec {v} {/ eq} и {eq} \ vec {w} {/ eq} — векторы, мы можем найти сумму двух векторов, {eq} \ vec {v} + \ vec {w} {/ eq}, построив параллелограмм, диагональ которого равна сумме.

Метод параллелограмма для вычитания векторов: Если {eq} \ vec {v} {/ eq} и {eq} \ vec {w} {/ eq} — векторы, мы можем найти разницу между двумя векторами, {eq} \ vec {v} — \ vec {w} {/ eq}, используя тот факт, что отрицательное значение вектора имеет противоположное направление исходного вектора и что добавление отрицательного элемента эквивалентно вычитанию {eq} (\ vec {v} — \ vec {w} = \ vec { v} + (- \ vec {w})) {/ экв}. Затем мы можем использовать тот же метод параллелограмма, что и для добавления векторов.

Давайте воспользуемся этими шагами и определениями, чтобы попрактиковаться в сложении или вычитании двух векторов с помощью метода параллелограмма со следующими двумя примерами.

Пример задачи 1 — сложение или вычитание двух векторов с помощью метода параллелограмма

Для {eq} \ vec {v} {/ eq} и {eq} \ vec {w} {/ eq} на диаграмме, которая из следующего показывает {eq} \ vec {v} + \ vec {w} {/ eq} получено методом параллелограмма?

Вариант 1)

Вариант 2)

Вариант 3)

Вариант 4)17

Шаг 1: При добавлении двух векторов {eq} \ vec {v} + \ vec {w} {/ eq}, нарисуйте два вектора так, чтобы у них была одна и та же начальная точка.Если вычесть два вектора, {eq} \ vec {v} — \ vec {w} {/ eq}, сначала измените направление второго вектора, {eq} \ vec {w} {/ eq}, чтобы создать {eq} — \ vec {w} {/ экв}. Затем нарисуйте {eq} \ vec {v} \ text {and} — \ vec {w} {/ eq}, чтобы у них была одна и та же начальная точка.

Поскольку мы имеем дело с добавлением двух векторов, мы рисуем векторы {eq} \ vec {v} \ text {и} \ vec {w} {/ eq}, чтобы у них была одна и та же начальная точка.

Шаг 2: Нарисуйте параллелограмм с двумя векторами в качестве двух смежных сторон.

Теперь мы создаем параллелограмм, в котором векторы {eq} \ vec {v} \ text {и} \ vec {w} {/ eq} — смежные стороны параллелограмма. Для этого мы рисуем линии, параллельные {eq} \ vec {v} {/ eq} и {eq} \ vec {w} {/ eq}, чтобы создать другие стороны параллелограмма.

Шаг 3: Нарисуйте вектор от начальной точки двух векторов к противоположной вершине параллелограмма, чтобы создать диагональ параллелограмма. Этот вектор представляет собой сумму или разность двух векторов.

Диагональ от начальной точки векторов до противоположной вершины параллелограмма является результирующим вектором, поэтому мы рисуем эту диагональ, чтобы получить наш вектор, который является суммой векторов {eq} \ vec {v} \ text {и} \ vec {w} {/ экв}.

Мы видим, что вариант 4 показывает правильный способ найти {eq} \ bf {\ vec {v} + \ vec {w}} {/ eq} с использованием метода параллелограмма.

Пример задачи 2 — сложение или вычитание двух векторов с помощью метода параллелограмма

Для {eq} \ vec {v} {/ eq} и {eq} \ vec {w} {/ eq} на диаграмме, которая из следующего показывает {eq} \ vec {v} — \ vec {w} {/ eq} получено методом параллелограмма?

Вариант 1)

Вариант 2)

Вариант 3)

Вариант 4)17

Шаг 1: При добавлении двух векторов {eq} \ vec {v} + \ vec {w} {/ eq}, нарисуйте два вектора так, чтобы у них была одна и та же начальная точка.Если вычесть два вектора, {eq} \ vec {v} — \ vec {w} {/ eq}, сначала измените направление второго вектора, {eq} \ vec {w} {/ eq}, чтобы создать {eq} — \ vec {w} {/ экв}. Затем нарисуйте {eq} \ vec {v} \ text {and} — \ vec {w} {/ eq}, чтобы у них была одна и та же начальная точка.

В этом случае мы вычитаем два вектора, поэтому сначала мы меняем направление {eq} \ vec {w} {/ eq}, чтобы получить {eq} — \ vec {w} {/ eq}, а затем нарисуйте два вектора так, чтобы они имели одинаковую начальную точку.

Шаг 2: Нарисуйте параллелограмм с двумя векторами в качестве двух смежных сторон.

Шаг 3: Нарисуйте вектор от начальной точки двух векторов к противоположной вершине параллелограмма, чтобы создать диагональ параллелограмма. Этот вектор представляет собой сумму или разность двух векторов.

Мы видим, что вариант 3 показывает правильный способ найти {eq} \ bf {\ vec {v} — \ vec {w}} {/ eq} с использованием метода параллелограмма.

Как сложить и вычесть векторы алгебраически

Введение

В этой статье мы рассмотрим вектор.Векторы — в отличие от простых чисел (скаляров), которые имеют только величину — имеют как величину (длину), так и направление. Мы рассмотрим, как представлять векторные величины, а также как их складывать и вычитать.

Ключевые термины

  • Скаляр
  • Вектор
  • Нулевой вектор
  • Цели

  • Количественное определение векторов с использованием системы координат
  • Сложение и вычитание векторов графически и алгебраически

  • Отдельные числа, то есть значения, имеющие только (положительную или отрицательную) величину, называются скалярами. Числа 0, –3, π, i, 1,3, e, и т. Д. — все это примеры скаляров. Другой тип значения, который часто используется в математике, — это вектор. Вектор — это величина, которая имеет направление как величины , так и направления . В этой статье мы рассмотрим некоторые математические характеристики векторов. Векторы имеют широкое применение, например, в физике.

    Введение в векторы

    Чтобы понять разницу между скаляром и вектором, полезно вспомнить физические примеры.Рассмотрим, например, температуру. Вы можете использовать термометр для измерения температуры воздуха в разных местах. В каждом случае вы получите некоторое число (и единицу) — скажем, 65 ° F. Это величина, но с ней не связано никакого направления; таким образом, это скалярная величина. Теперь рассмотрим измерения ветра в тех же местах. Когда вы измеряете ветер, вы, вероятно, измеряете и скорость, и направление. Таким образом, ваши измерения ветра составляют вектор. Мы могли бы выразить этот вектор в виде стрелки, указывающей в направлении ветра, причем длина стрелки пропорциональна скорости ветра.Ниже показаны два измерения ветра, сделанные в разных точках; стрелки представляют векторы, связанные с этими измерениями.

    Векторы имеют величину и направление, но сами по себе не имеют назначенного местоположения. То есть, пока сохраняется направление и длина «стрелки», мы можем перемещать ее куда угодно, не меняя ее. Это важная характеристика, которая позволит нам активно работать с векторами.

    Представление векторов

    Наша первая задача — найти способ четко и последовательно представлять векторы. Графически это просто: поскольку мы можем перемещать вектор куда угодно, давайте всегда располагаем «хвост» вектора в начале координатной плоскости. (Обратите внимание, что «голова» и «хвост» вектора определены, как показано ниже.)

    Теперь, когда хвост вектора помещен в начало координат (помните, мы можем перемещать вектор куда угодно, если сохраняем его направление и длину), мы можем количественно определить его как координаты головы.Пример показан ниже для вектора v . (Обратите внимание: чтобы отличать символы, представляющие векторы, от символов, представляющих скаляры, мы используем полужирный шрифт. Другой распространенный метод — использовать небольшую стрелку над символом: например, вектор.)

    Таким образом, вектор v — это просто координаты точки в (2, 3). Обратите внимание, что все векторы, показанные ниже, равны (2, 3) — наше соглашение заключается в том, что вектор описывается координатами точки в его голове только , когда его хвост расположен в начале координат.

    Хотя мы показали вектор только в двух измерениях, этот подход можно обобщить на любое количество измерений. Например, в трех измерениях вектор будет иметь вид ( x, y, z ). Все свойства двумерных векторов можно легко расширить до трех измерений.

    Но как нам «переместить» вектор с числовой точки зрения? Например, скажем, вектор v имеет голову в (3, 2) и хвост в (1, 4).


    Ответ заключается в переводе (или перемещении) головы и хвоста на эквивалентное расстояние и в одном направлении. Это преобразование должно привести к тому, что хвост вектора переместится в начало координат — простой процесс, который включает вычитание каждой координаты хвоста из себя. В приведенном выше примере результат (3–3, 2–2) = (0, 0). Чтобы перевести голову, аналогичным образом вычтите координаты хвоста из координат головы — это удовлетворяет нашему критерию, что перевод имеет фиксированное расстояние и направление.Таким образом, голову нужно двигать следующим образом: (1 — 3, 4 — 2) = (–2, 2). Таким образом, в общем, чтобы найти значение произвольно расположенного вектора, вычтите координаты хвоста из координат головы. Этот процесс проиллюстрирован ниже.

    Обратите внимание, что вектор (0, 0), иногда называемый нулевым вектором , имеет длину 0, но не имеет определенного направления. (То есть независимо от того, какое направление вы выберете, нулевой вектор будет одинаковым.)

    Практическая задача: Определите значение каждого вектора, показанного на графике ниже.

    Решение: В каждом случае можно найти координатное выражение для вектора, вычитая координаты хвоста из соответствующих координат головы. Это работает, даже если хвост находится в начале координат, имеющем координаты (0, 0). Но если хвост находится в начале координат, вектор также просто равен координатам головы. Если это вам поможет, перерисуйте векторы так, чтобы хвосты располагались в начале координат.

    a = (–1, 4)

    b = (–3, –3)

    c = (3 — 3, 2 — 0) = (0, 2)

    d = (3 — 2, –4 — [–1]) = (1, –3)

    Сложение и вычитание векторов


    Как и в случае со скалярами, мы можем складывать и вычитать векторы. Процесс аналогичен, но с одной или двумя оговорками. Чтобы сложить или вычесть два вектора a и b , добавьте или вычтите соответствующие координаты вектора.То есть, где a и b определены следующим образом, вот правила сложения и вычитания.


    Обратите внимание, что, как и в случае со скалярами, сложение векторов коммутативно, а вычитание — нет. Графически мы складываем два вектора a и b , помещая хвост b в голову a , а затем создавая новый вектор, начиная с хвоста a и заканчивая в голове b. .Координаты этого нового вектора определяются так же, как и раньше: путем размещения его хвоста в начале координат. Этот процесс проиллюстрирован ниже для векторов a = (4, 1) и b = (-1, 2).

    Обратите внимание, что

    Вычитание векторов следует в основном той же процедуре, что и сложение, за исключением того, что вычитаемый вектор «меняет направление» на противоположное.Рассмотрим те же векторы a и b , как указано выше, за исключением того, что мы вычислим a, b. (Обратите внимание, что это то же самое, что и , где — b имеет ту же длину, что и b , но имеет противоположное направление.)

    Практическая задача: Выполните следующие векторные операции.

    а. (3, 2) — (4, 5) б.(-1, 5) + (10, -6) с. (-1, 0) — (0, 0)

    Решение: В каждом случае сложите или вычтите соответствующие координаты, чтобы найти результат. Один из полезных способов проверить свой ответ — нарисовать векторы на графике, показывая сложение или вычитание и сравнивая ваши результаты.

    а. (-1, -3) б. (9, -1) с. (-1, 0)

    Добавление векторов — определение, метод сложения, формула, закон и часто задаваемые вопросы

    Мы не можем сложить два вектора, чтобы получить результат, поскольку они имеют величину, а также направление.Сложить скалярную величину очень просто, но в случае векторов это немного сложно.

    Чтобы понять разницу и лучше изучить, предположим, что автомобиль движется на 10 миль к северу и 10 миль к югу. Мы можем легко определить общее расстояние, пройденное на машине, сложив эти два числа, например, 20 миль. Но в случае сложения векторов результат нулевой.

    Причина в том, что северное и южное направления противоположны друг другу, поэтому они уравновешиваются, и поэтому векторная сумма будет равна нулю.В этой статье дается ясный вывод о сложении двух векторов, или, можно сказать, о «векторной сумме».

    Сумма двух векторов

    Рассмотрим два вектора \ [\ overrightarrow {u} \] и \ [\ overrightarrow {v} \]. Мы собираемся добавить соответствующие компоненты. Напишем о компонентах векторов:

    \ [\ overrightarrow {u} \] = ⟨u1, u2⟩ и \ [\ overrightarrow {v} \] = ⟨v1, v2⟩

    Когда мы делаем суммирование векторов выше, результат будет:

    \ [\ overrightarrow {u} \] + \ [\ overrightarrow {v} \] = ⟨u1 + v1, u2 + v2⟩

    Суммирование двух векторов может быть вызвано как результат.

    Формула сложения векторов

    Существует два типа методов сложения векторов:

    1. Закон векторов треугольника

    2. Закон векторов параллелограмма

    Как сложить два вектора?

    Вы все еще не знаете, как складывать векторы?

    Вот несколько советов, которые следует помнить при сложении векторов:

    • Сложение векторов выполняется геометрически, но не алгебраически.

    • Векторные величины перед суммированием должны вести себя как независимые друг от друга.

    • Из сложения векторов мы делаем вывод только о результирующем количестве векторов, распространяемых по телу.

    • Из сложения векторов мы получаем результирующий вектор, который не зависит от суммирования векторов как \ [\ overrightarrow {A} \] + \ [\ overrightarrow {B} \] = \ [\ overrightarrow {B } \] + \ [\ overrightarrow {A} \].

    Закон сложения векторов треугольников

    При рассмотрении треугольников сложение векторов является зависимым.Теперь нам нужно выяснить, как это работает.

    Предположим, что \ [\ overrightarrow {a} \] и \ [\ overrightarrow {b} \] — два вектора.

    Здесь вам нужно провести линию AB, которая называется \ [\ overrightarrow {a} \] хвостом с A и \ [\ overrightarrow {b} \] с головой. Давайте нарисуем линию BC, которая выделяет \ [\ overrightarrow {b} \] с B в качестве конца и C в качестве головы.

    Давайте закончим треугольник, проведя линию AC с A на конце и C на короне. Сумма двух векторов \ [\ overrightarrow {a} \] & \ [\ overrightarrow {b} \] представлена ​​линией AC.{2} + 2abcos⁡θ} \]

    Здесь

    Величина вектора = a

    Величина вектора = b

    θ — это угол, охватываемый вектором \ [\ overrightarrow {a} \] & vector \ [\ overrightarrow {b} \]

    Считайте, что равнодействующая векторов составляет угол ф с \ [\ overrightarrow {a} \]; тогда выражение будет:

    tan⁡ф = \ [\ frac {bsin⁡θ} {a + bcos⁡θ} \]

    Нам нужно изучить это с помощью примера. Предположим, у нас есть два вектора с равной величиной A, а θ — угол между этими двумя векторами.{2} + 2AAcos⁡θ} \] = 2Acos⁡ \ [\ frac {θ} {2} \]

    Считайте, что равнодействующая векторов составляет угол ф с \ [\ overrightarrow {a} \]; тогда выражение будет:

    tan⁡ф = \ [\ frac {A sin⁡θ} {A + A cos⁡θ} \] = tan tan \ [\ frac {θ} {2} \]

    Тогда , ф = \ [\ frac {θ} {2} \]

    Закон сложения векторов параллелограмма

    Мы также можем понять концепцию сложения векторов, используя закон параллелограмма.

    Добавить комментарий

    Ваш адрес email не будет опубликован. Обязательные поля помечены *